Anda di halaman 1dari 47

Professional Responsibility Outline

1. The Regulation of Lawyers: Historical Background and an Overview of Bar Admissions, Professional
Discipline, and Judicial Sanctions. Casebook, pp.1-16, 31-82
a) Restatement Third, The Law Governing Lawyers§ 2, Comment d
i) ―A license to practice law confers great power on lawyers to do good or wrong. Lawyers practice an
occupation that is complex and often, particularly to nonlawyers, mysterious. Clients and others are
vulnerable to wrongdoing by corrupt lawyers. Hence, as far back as the first bars in medieval
England efforts have been made to screen candidates for the bar with respect to their character. * * *
The central inquiry concerns the present ability and disposition of the applicant to practice law
competently and honestly.‘‘
ii) Concerns about how bar committees might enforce this?
-Communist party membership
-In re: Hale (Ill., 1999) (outspoken racist)
-In re: Converse (Neb., 1999) (law school weirdo)
iii) Rule 8.4 Professional Misconduct:
It is professional misconduct for a lawyer to:
(a) violate or attempt to violate the Rules of Professional Conduct, knowingly assist or induce
another to do so, or do so through the acts of another;
(b) commit a criminal act that reflects adversely on the lawyer's honesty, trustworthiness or
fitness as a lawyer in other respects;
(c) engage in conduct involving dishonesty, fraud, deceit or misrepresentation;
 Comment 2:
o ―A lawyer should be professionally answerable only for offenses that indicate lack of
those characteristics relevant to law practice.‖
o -violence, dishonesty, breach of trust, or serious interference with the administration of
justice.
o -not crimes of ordinary ―moral turpitude‖: adultery, etc.
iv) Rule 8.1 Bar Admission and Disciplinary Matters
An applicant for admission to the bar, or a lawyer in connection with a bar admission application
or in connection with a disciplinary matter, shall not:
(a) knowingly make a false statement of material fact; or
(b) fail to disclose a fact necessary to correct a misapprehension known by the person to have
arisen in the matter…except that this Rule does not
1. require disclosure of information otherwise protected by Rule 1.6.
v) Should there be a statute of limitations on how long prior incidents can affect current bar admission
decisions? Matter of Prager (Mass, 1996) – not enough lapsed time to admit reformed druglord to
bar.
 ABA Standards for Imposing Lawyer Sanctions, approved by 1986 ABA House of Delegates
(amended in 1992). [adapted by many states]
o Four factors:
1. The duty violated
2. The lawyer‘s mental state
3. The seriousness of the actual or potential injury
4. The existence of aggravating or mitigating factors
vi) Why discipline attorneys? ABA Standards: ―The purpose of lawyer discipline proceedings is to
protect the public and the administration of justice from lawyers who have not discharged, will not
discharge, or are unlikely to properly discharge their professional duties to clients, the public, the
legal system, and the legal profession.‖
1
(a) Protects clients and other non-attorneys
(b) Removes seriously deviant lawyers
(c) Deters deviance and maximizes compliance with rules
(d) Forestalls public dissatisfaction, protects the public image of the profession.
(e) Not punishment, per se.
(f) Not the equivalent of a criminal sanction.
vii) Model Rule 8.3
A lawyer who knows that another lawyer has committed a violation of the Rules of Professional
Conduct that raises a substantial question as to that lawyer‘s honesty, trustworthiness or fitness as a
lawyer in other respects, shall inform the appropriate professional authority.
In re Hinson-Lyles (34) – three felonies for sex with students sufficient to deny.

(a) Exceptions?
(i) -(Rule 1.6)
(ii) -But what about Himmel (pg 56) – first case to discipline lawyer solely for failure to
report.
viii) Model Rule 1.1
A lawyer shall provide competent representation to a client.
 Competent representation requires the legal knowledge, skill, thoroughness and preparation
reasonably necessary for the representation.
 Comments:
(1) Sometimes general practitioner competence is enough. Sometimes expertise in a
particular field may be required. Relevant factors for determining whether a lawyer has
the requisite knowledge and skill include:
1. How complex or specialized is the matter?
2. What is the lawyer‘s experience (in general and in the specific field)
3. Does the lawyer have time to prepare and study?
4. Can the lawyer refer matter to, or associate or consult with, a lawyer of established
competence in field?
(2) A lawyer need not necessarily have special training or experience to handle unfamiliar
types of legal problems.
(3) Emergencies = different standards.
(4) Can accept case if you can achieve competence through reasonable preparation.
(5) Inquiry into and analysis of the factual and legal elements; methods and procedures.
(6) Continuing legal education: must keep abreast of changes in the law and its practice.
 Examples of Incompetence
(1) Not understanding basic legal rules and principles
1. not knowing the necessary procedures to complete an adoption
2. Misadvising client about basic tax and bankruptcy rules, etc.
3. Failure to name necessary parties in a lawsuit
4. Not knowing local court rules re: filings and service
(2) Lack of preparation
1. Not conducting necessary legal research
2. Filing sloppy, incoherent briefs
ix) Model Rule 1.3
A lawyer shall act with reasonable diligence and promptness in representing a client.
 Types of 1.3 Violations:
(i) Missing deadlines; statutes of limitations.
(ii) Unreasonable delay
(iii)Neglect
2
1. ―indifference and a consistent failure to carry out the obligations which the lawyer
has assumed…or conscious disregard for the responsibility owed‖
2. Note: Malpractice v. Professional Discipline
(iv) Inadequate investigation work
 In Conduct of Loew (Or. 1982): burn-out syndrome is an excuse
x) What if two states have incompatible rules (i.e. one requires disclosure and the other forbids it)?
Which jurisdiction can discipline the attorney? Which rules?
Rule 8.5(b) Choice of Law.
In any exercise of the disciplinary authority of this jurisdiction, the rules of professional conduct to
be applied shall be as follows:
(1) for conduct in connection with a matter pending before a tribunal, the rules of the
jurisdiction in which the tribunal sits, unless the rules of the tribunal provide otherwise;
and
(2) for any other conduct, the rules of the jurisdiction in which the lawyer‘s conduct
occurred, or, if the predominant effect of the conduct is in a different jurisdiction, the
rules of that jurisdiction shall be applied to the conduct. A lawyer shall not be subject to
discipline if the lawyer‘s conduct conforms to the rules of a
i. jurisdiction in which the lawyer reasonably believes the predominant effect of
the lawyer‘s conduct will occur.
xi) Regulating lawyers outside of the formal disciplinary system. By what standard should a lawyer be
judged in a malpractice case?
 Restatement Third, The Law Governing Lawyers § 52(1): ‗‗[A] lawyer who owes a duty of
care must exercise the competence and diligence normally exercised by lawyers in similar
circumstances.‘‘
 ‗‗[T]he essential elements of a cause of action for professional negligence are:
‗‗(1) The employment of the attorney or other basis for imposing a duty;
‗‗(2) the failure of the attorney to exercise ordinary skill and knowledge; and
‗‗(3) that such negligence was the proximate cause of damage to the plaintiff.‘‘
-Ronald E. Mallen & Jeffrey M. Smith, Legal Malpractice § 8.13 (2010 ed.).
 Some courts loosening up on the ―proximate cause‖ rule: ―substantial factor‖ of plaintiff‘s
damages may be enough.
 Wide range of potential penalties: compensatory damages (including business costs), punitive
damages, contempt charges, disqualification, and fee forfeiture.
xii) Medical Malpractice Case.
 PP4, Field is likely liable to Moore because:
 Failure to have client examined by independent physician does not demonstrate ―the
competence and diligence normally exercised.‖
 Because of her failure to do so, she had no reasonable basis to advise Moore to accept the
settlement.
xiii) Tax Case
 Was Field obliged to refer the tax case to a specialist? Almost certainly.
(1) Horne v. Peckham - told client that he had no knowledge of tax matters. Still liable for
malpractice.
1. Duty to refer client to a specialist or recommend the assistance of a specialist if a
reasonably careful and skillful practitioner would do so.
 Lucas v. Hamm ―Rule against perpetuities‖ case will almost never save you
 On unsettled points of law, reasonable judgment may not be the basis of malpractice claim,
even if it ultimately proves to be wrong
xiv) Proving malpractice claims

3
 Plaintiff must ordinarily present expert testimony about duty of care.
 Evidence must be presented on the record with chance for cross examination (unless
absolutely obvious).
 Experts can cite the model rules as evidence of standards, but the model rules are not
determinative.
 Some courts have argued that violations of professional rules create rebuttable presumptions
of malpractice.
xv) Non-negligence malpractice claims
For non-negligence breaches (e.g. breach of duty of loyalty), lawyers are subjected to fee forfeiture,
even if client does not show actual damage.
 (Hendry v Pelland (DC Cir 1996)) - Firm hired by son to prevent sale of property on terms
agreed to by mother; eventually represented all family members: son, mother, infant
grandchildren).
 Because Court found impossible conflicting interests, firm was denied fees from any of these
clients.
xvi) Liability to Non-Clients
At least 4 situations (Restatement):
1) Negligence or lack of loyalty to prospective clients
2) Beneficiaries named in a client‘s will.
3) non-client to whom lawyer has expressly assumed obligation to accurately report
information (e.g.insurance cases)
4) Aiding a client breach their fiduciary obligation to a third-party intended beneficiary.
2. Prospective Clients. (pp5) Casebook, pp.84-97
a) Morality
viii) Does a lawyer makes an implicit moral decision every time she takes a case?
ix) Monroe Freedman: Yes.
 Because you are not bound to take cases, you can be morally accountable.
x) ABA: No.
xi) Rule 1.2(b):
A lawyer‘s representation of a client…does not constitute an endorsement of the client‘s
political, economic, social or moral views or activities.
Comment 5: Legal representation should not be denied to people who are unable to afford legal
services, or whose cause is controversial or the subject of popular disapproval. By the same
token, representing a client does not constitute approval of the client's views or activities.
b) Duty to Prospective Clients
viii) Rule 1.18(b)
Even when no client-lawyer relationship ensues, a lawyer who has had discussions with a
prospective client shall not use or reveal information learned in the consultation, except as Rule 1.9
would permit with respect to information of a former client.
 Potential malpractice liability for bad advice.
c) Formation of Client Relationship (pp5)
Restatement Third, The Law Governing Lawyers § 14:
‗‗A relationship of client and lawyer arises when:
‗‗(1) a person manifests to a lawyer the person‘s intent that the lawyer provide legal services for
the person; and either
‗‗(a) the lawyer manifests to the person consent to do so;
or

4
‗‗(b) the lawyer fails to manifest lack of consent to do so and the lawyer knows or
reasonably should know that the person reasonably relies on the lawyer to provide the
services
Rule 1.18(c)
A lawyer subject to paragraph (b) shall not represent a client with interests materially adverse
to those of a prospective client in the same or substantially related matter if the lawyer received
information from the prospective client that could be significantly harmful to that person in the
matter, except as provided in paragraph (d). If a lawyer is disqualified from representation under
this paragraph, no lawyer in a firm with which that lawyer is associated may knowingly undertake or
continue representation in such a matter, except as provided in paragraph (d).

Rule 1.18(d)
(d) When the lawyer has received disqualifying information as defined in paragraph (c), representation
is permissible if:
(1) both the affected client and the prospective client have given informed consent, confirmed in
writing, or:
(2) the lawyer who received the information took reasonable measures to avoid exposure to more
disqualifying information than was reasonably necessary to determine whether to represent the
prospective client; and
(i) the disqualified lawyer is timely screened from any participation in the matter and is
apportioned no part of the fee therefrom; and
(ii) written notice is promptly given to the prospective client.
d) Nonprofit limited legal service programs
Imagine that, instead of meeting Mr. Cannell in your office, you had talked to him during the pro bono
work that you do at a local legal services agency. Suppose that Mr. Cannell did not want to hire you; he
simply wanted to be given some basic advice and told how to file a claim against the broker himself.
Under those circumstances, would Mr. Cannell have been a prospective client or an actual client?
 Rule 6.5 seems to treat Mr. Cannell as an actual client, not a prospective client. The qualifier is
that it is not a ―continuing‖ relationship.
 Therefore, no imputed disqualification.

 Comment 4: By virtue of paragraph (b), however, a lawyer's participation in a short-term limited
legal services program will not preclude the lawyer's firm from undertaking or continuing the
representation of a client with interests adverse to a client being represented under the program's
auspices. Nor will the personal disqualification of a lawyer participating in the program be
imputed to other lawyers participating in the program.

3. Attorney Billing Issues and Contingency Fees. Casebook, pp.98-115


a) Model Rule 1.5 (a)
(a) A lawyer shall not make an agreement for, charge, or collect an unreasonable fee or an unreasonable
amount for expenses. The factors to be considered in determining the reasonableness of a fee include the
following:
(1) the time and labor required, the novelty and difficulty of the questions involved, and the skill
requisite to perform the legal service properly;
(2) the likelihood, if apparent to the client, that the acceptance of the particular employment will
preclude other employment by the lawyer;
(3) the fee customarily charged in the locality for similar legal services;
(4) the amount involved and the results obtained;
(5) the time limitations imposed by the client or by the circumstances;
(6) the nature and length of the professional relationship with the client;
5
(7) the experience, reputation, and ability of the lawyer or lawyers performing the services; and
(8) whether the fee is fixed or contingent.
b) Model Rule 1.5 (b)
(b) The scope of the representation and the basis or rate of the fee and expenses for which the client
will be responsible shall be communicated to the client, preferably in writing, before or within a
reasonable time after commencing the representation, except when the lawyer will charge
a regularly represented client on the same basis or rate. Any changes in the basis or rate of the fee or
expenses shall also be communicated to the client.
c) Model Rule 1.5 (c)
(c) A fee may be contingent on the outcome of the matter for which the service is rendered, except in a
matter in which a contingent fee is prohibited by paragraph (d) or other law. A contingent fee agreement
shall be in a writing signed by the client and shall state the method by which the fee is to be
determined, including the percentage or percentages that shall accrue to the lawyer in the event of
settlement, trial or appeal; litigation and other expenses to be deducted from the recovery; and whether
such expenses are to be deducted before or after the contingent fee is calculated. The agreement must
clearly notify the client of any expenses for which the client will be liable whether or not the client is the
prevailing party.
d) Nonrefundable retainers are generally NOT permissible
 Clients do not have to pay for legal services they have not received
• It is still the client‘s money: Retainers and flat fee payments must be placed in a trust fund until
they have actually been earned.
 Exceptions:
•lawyer can demonstrate opportunity costs (that had been foreseeable to client)
•Fee arrangement proposed by sophisticated clients (sometimes).
- Raymark Industries: client proposed million dollar fixed fees; tried to fire firm 10
weeks into work.
e) Improper Billing
• Nonlawyer work at lawyer rates.
– Faxing, making basic phone calls, delivering documents, being a friend.
Ex. Green (Colo 2000): fees for time lawyer spent faxing documents found excessive.
• Real world: this happens every day.
Rule 1.5 (comment 1): you can bill for it, but it has to be part of billing arrangement.
• Charging for time necessitated by own inexperience.
– excessive hours doing research or consulting with other attorneys

f) Concerns about Contingency Fees


 Attorney windfalls
•Minimal relationship between effort and skill of attorney andpayment.
– Alternative proposal: contingency based on gap between initial settlement offer and
ultimate result?
 Incentives to ―play the litigation lottery‖? (this is largely a red herring)
 Prohibitions
• Domestic relations cases: May discourage reconciliation. Rule 1.5 (d)
•Criminal defense cases: May discourage guilty pleas to lesser charges. . Rule 1.5 (d)
•Civil defense cases:
– ABA formal opinion found them permissible. Some courts have disagreed.
c) – Practical issue: measuring the contingency.
g) Model Rule 1.5(d)
A lawyer shall not enter into an agreement for, charge, or collect:

6
(1) any fee in a domestic relations matter, the payment or amount of which is contingent upon the
securing of a divorce or upon the amount of alimony or support or property settlement in lieu thereof; or
(2) a contingent fe for representing a defendant in a criminal case
h) Hourly Billing Issues
• Creates incentives for firm to perform unnecessary work, or to lie about work.
– Particularly in larger firms (pressure to bill)
– Attorneys dealing with unsophisticated or desperate clients
 Leads to absurd results
o $30,000 memos
• Quality of life concerns
– Especially for mothers
• Alternative Trend: Fixed fees for ―commodity work‖

3. Handling Client Property and Withdrawing from Representation. Casebook, pp.115-131


 Rule 1.15(a):
A lawyer shall hold property of clients or third persons that is in a lawyer‘s possession in connection
with a representation separate from the lawyer‘s own property.

 Restatement § 44, Comment e:


‗‗This Section requires a lawyer to use reasonable measures for safekeeping such objects, for example,
by placing them in a safe-deposit box or office safe. The reasonableness of measures depends on the
circumstances, including the market value of the property, its special value to the client * * *, and special
difficulties that would be required to replace it if known to the lawyer * * *.‘‘
 Grosso, 760 So.2d 940 (Fla.2000) (client‘s guns in humid garage)
 PP7 - Did Jackson violate the Model Rules by signing the check? •Not on these facts.
o •Lawyers need client permission to sign clients‘ names, but can sign their own.
 • Otherwise, possible disciplinary and criminal liability (conversion)
 Model Rule 1.15: Safekeeping Property
a) A lawyer shall hold property of clients or third persons that is in a lawyer‘s possession in connection
with a representation separate from the lawyer’s own property. Funds shall be kept in a separate
account maintained in the state where the lawyer‘s office is situated, or elsewhere with the consent of
the client or third person. Other property shall be identified as such and appropriately safe-guarded.
(d) Upon receiving funds or other property in which a client or third person has an interest, a lawyer
shall promptly notify the client or third person. Except as stated in this rule or otherwise permitted
by law or by agreement with the client, a lawyer shall promptly deliver to the client or third person any
funds or other property that the client or third person is entitled to receive...
Comments:
o [1] A lawyer should hold property of others with the care required of a professional
fiduciary. Securities should be kept in a safe deposit box, except when some other form
of safekeeping is warranted by special circumstances. All property that is the property of
clients or third persons, including prospective clients, must be kept separate from the
lawyer’s business and personal property and, if monies, in one or more trust accounts.
o [3] Lawyers often receive funds from which the lawyer‘s fee will be paid. The lawyer is
not required to remit to the client funds that the lawyer reasonably believes
represent fees owed. However, a lawyer may not hold funds to coerce a client into
accepting the lawyer‘s contention. The disputed portion of the funds must be kept in a
trust account and the lawyer should suggest means for prompt resolution of the dispute,
such as arbitration. The undisputed portion of the funds shall be promptly
distributed.

7
 Fiduciary responsibilities regarding client property
 No intent or harm required for discipline
 • One of the easiest ways to get disbarred
 • Property includes client files (AZ, MO state bar
 opinions); emails & electronic files (NY opinion)
 • Lawyer needs individual records of each client‘s funds.
 • ABA Model Rules for Client Trust Accounts
o • Detailed ledger records, 5 year requirement
 RULE 1.16
 (a) Except as stated in paragraph (c), a lawyer shall not represent a client or, where
representation has commenced, shall withdraw from the representation of a client if:
(1) the representation will result in violation of the rules of professional conduct or other
law;
(2) the lawyer‘s physical or mental condition materially impairs the lawyer‘s ability to
represent the client; or
(3) the lawyer is discharged.
 (b) Except as stated in paragraph (c), a lawyer may withdraw from representing a client if:
(1) withdrawal can be accomplished without material adverse effect on
the interests of the client;
(2) the client persists in a course of action involving the lawyer‘s
services that the lawyer reasonably believes is criminal or fraudulent;
(3) the client has used the lawyer‘s services to perpetrate a crime or
fraud;
(4) the client insists upon taking action that the lawyer considers
repugnant or with which the lawyer has a fundamental disagreement;
(5) the client fails substantially to fulfill an obligation to the lawyer
regarding the lawyer's services and has been given reasonable warning
that the lawyer will withdraw unless the obligation is fulfilled;
(6) the representation will result in an unreasonable financial burden
on the lawyer or has been rendered unreasonably difficult by the
client; or
(7) other good cause for withdrawal exists.
 (d) Upon termination of representation, a lawyer shall take steps to the extent reasonably
practicable to protect a client's interests, such as giving reasonable notice to the client,
allowing time for employment of other counsel, surrendering papers and property to which the
client is entitled and refunding any advance payment of fee or expense that has not
been earned or incurred. The lawyer may retain papers relating to the client to the extent
permitted by other law.

4. Privilege and Confidentiality. Casebook, pp.131-148


1) Attorney-Client Privilege
a. •Communication between privileged persons, in confidence, for the purpose of legal assistance.
(Restatement §68; FRE 503)
– Expansive, with some exceptions
•Source of considerable litigation
b. Loss of Attorney Client Privilege
 A-C privilege is waived if the client, the client‘s lawyer, or another authorized agent of the
client voluntarily discloses the communication in a non-privileged communication (see
Restatement 79).
8
o – Most common scenario: client shares information with outsider
o – A-C privilege can be waived through accidental disclosure (a real concern in large
document-intensive production cases).
 • Once A-C privilege is lost, possible full subject matter waiver (same regarding work product
waiver).
 Not automatically lost with client‘s death
o – Swidler & Berlin v US (1998) (Vince Foster)
(1) Restatement 77 disagrees with this. Should be a balancing test.
 Despite loss of A-C privilege, confidentiality duty remains
o – When A-C privilege waived (like ineffective assistance of counsel claim), lawyer must still
protect confidentiality from prosecutor.
Attorney Work product:
 –Tangible material or its equivalent in unwritten or oral form, other than underlying facts,
prepared by a lawyer for litigation then in progress or in reasonable anticipation of future
litigation.
 –Not absolute: ‖substantial need‖ and ―undue hardship‖ showing
 –Courts grant special protection to ―core‖ or ―opinion‖ work product (―extraordinary
circumstances‖ standard).
o •―not even the most liberal of discovery theories can justify unwarranted inquiries into
the files and the mental impressions of an attorney.‖ (Hickman)
2) Duty of Confidentiality
Model Rule 1.6:
(a) A lawyer shall not reveal information relating to the representation of a client unless the client gives
informed consent, the disclosure is impliedly authorized in order to carry out the representation or the
disclosure is permitted by paragraph (b).
 Comment 3:
The confidentiality rule…applies not only to matters communicated in confidence by the client but
also to all information relating to the representation, whatever its source.
Extra: Marzan: duty remains even if information is ―publicly available‖ (but not ―generally
known.‖) This is broader than A-C privilege.
(b)A lawyer may reveal information relating to the representation of a client to the extent the lawyer
reasonably believes necessary:
(1) to prevent reasonably certain death or substantial bodily harm;
(2) to prevent the client from committing a crime or fraud that is reasonably
certain to result in substantial injury to the financial interests or property of
another and in furtherance of which the client has used or is using the lawyer‘s
services;
(3) to prevent, mitigate or rectify substantial injury to the financial interests or
property of another that is reasonably certain to result or has resulted from the
client‘s commission of a crime or fraud in furtherance of which the client has used
the lawyer‘s services;
(4) to secure legal advice about the lawyer‘s compliance with these Rules;
(5) to establish a claim or defense on behalf of the lawyer in a controversy
between the lawyer and the client, to establish a defense to a criminal charge or
civil claim against the lawyer based upon conduct in which the client was
involved, or to respond to allegations in any proceeding concerning the lawyer‘s
representation of the client; or
(6) to comply with other law or a court order.
 Comment 2: The Policy Rationale
9
A fundamental principle in the client-lawyer relationship is that, in the absence of the client‘s
informed consent, the lawyer must not reveal information relating to the representation. See Rule
1.0(e) for the definition of informed consent. This contributes to the trust that is the hallmark of the
client-lawyer relationship. The client is thereby encouraged to seek legal assistance and to
communicate fully and frankly with the lawyer even as to embarrassing or legally damaging subject
matter. The lawyer needs this information to represent the client effectively and, if necessary, to
advise the client to refrain from wrongful conduct. Almost without exception, clients come to
lawyers in order to determine their rights and what is, in the complex of laws and regulations,
deemed to be legal and correct. Based upon experience, lawyers know that almost all clients follow
the advice given, and the law is upheld.
 Comment 4
This prohibition also applies to disclosures by a lawyer that do not in themselves reveal
protected information but could reasonably lead to the discovery of such information by a
third person. A lawyer‘s use of a hypothetical to discuss issues relating to the representation is
permissible so long as there is no reasonable likelihood that the listener will be able to ascertain the
identity of the client or the situation involved.
3) Differences between A-C privilege, work product immunity, and confidentiality duty
 Sources: evidence law, civil procedure law, agency law.
• Purposes: client trust; attorney diligence and preparation
• Scope of Protection: confidentiality extends beyond legal proceedings.
• A-C: absolute; WP: subject to necessity exception.

5. Organizational Clients. Casebook, pp.148-167


a. Upjohn (1981): rejected ―control group‖ test. Standard– ―The communications concerned matters
within the scope of the employees corporate duties, and the employees themselves were sufficiently
aware that they were being questioned in order that the corporation could obtain legal advice.‖
b. Rule 1.13 –Organization as a client
 •Privilege is the corporation‘s to waive [not either group of managers, technically, but successor
management can clearly see the report]; shareholders bringing a derivative suit may have access too.
 •Unlike in individual representation, this is not necessarily for the person who hired you or the person
most at risk to decide.
c. Investigations and Waiver
 Should the attorney-client privilege and work product immunity be lost as to later private
litigation if a company voluntarily turns documents over to a government agency in an effort to
demonstrate there has been no violation of law?
 Hot button issue due to federal enforcement efforts in the 2000s.
o •Post-Enron, WorldCom, Adelphia, Tyco, HealthSouth fall-out; emphasis
on punishing bad corporate actors and deterring them by any means
necessary.
o •Pressure to waive; credit for doing so
 In re: Columbia/HCA Healthcare Corp (6th Cir, 2002): turned over privileged
matter to gov‘t with non-waiver statement (ex. This does not waive the
confidential status of this); Court found that this waived A-C and work product
protection in later civil litigation.
 Corporate criminal defendants got credit for waiving/disclosing. ABA and local
bar associations vehemently opposed this. Now this is supposedly no longer a
consideration.

10
 Limited waiver – turning over information to government to prove innocence. Only
waiving privilege to government and no one else. No one else should be able to see/use
government.
 Putting aside A-C privilege and WP issues, would your duty of confidentiality under Model Rule
1.6 prevent you from voluntarily turning the memorandum over to the EPA?
 Attorney is clearly allowed to disclose (1.6(b)).
 1.6, Comment 6:
Paragraph (b)(1) recognizes the overriding value of life and physical integrity and permits
disclosure reasonably necessary to prevent reasonably certain death or substantial bodily
harm. Such harm is reasonably certain to occur if it will be suffered imminently or if there is
a present and substantial threat that a person will suffer such harm at a later date if the lawyer
fails to take action necessary to eliminate the threat. Thus, a lawyer who knows that a client
has accidentally discharged toxic waste into a town‘s water supply may reveal this
information to the authorities if there is a present and substantial risk that a person who
drinks the water will contract a life threatening or debilitating disease and the lawyer‘s
disclosure is necessary to eliminate the threat or reduce the number of victims.
 Is A-C privilege and WP immunity lost in later litigation as to documents a company turns over to its
outside accounting firm as part of the company‟s annual audit?(PP9)
a) General rule: Work product protection not waived if given to non-adversaries (unlike A-C
privilege).
 Does either the attorney-client privilege or work product immunity protect the memorandum from
the former assistant general counsel that purports to authorize false reporting?
a) •The Crime-fraud exception (see Restatement 82)
a. •NOTE: This is different than Rule 1.6(b)
 Lawyer might have affirmative obligation to disclose to avoid assisting fraud.
Rule 4.1
In the course of representing a client a lawyer shall not knowingly:
(a) make a false statement of material fact or law to a third person; or
(b) fail to disclose a material fact when disclosure is necessary to avoid assisting a criminal or
fraudulent act by a client, unless disclosure is prohibited by Rule 1.6.
- Common Interest Privilege
o If a single lawyer represents more than one defendant, she is likely to get confidential
information from each.

6. The Duty of Loyalty: Conflicts of Interest, Direct Adversity, and Waiver. Casebook,
pp.168-201, PP 10
a) Model Rule 1.7(a)
‗‗A concurrent conflict of interest exists if:
(1) the representation of one client will be directly adverse to another client; or
(2) there is a significant risk that the representation of one or more clients will be materially limited by
the lawyer’s responsibilities to another client, a former client or a third person or by a personal interest
of the lawyer.‘‘ [emphasis added]
 see also, Restatement § 121
b) Model Rule 1.7(b)
(b) Notwithstanding the existence of a concurrent conflict of interest under paragraph (a), a lawyer may
represent a client if:
(1) the lawyer reasonably believes that the lawyer will be able to provide competent and diligent
representation to each affected client;
(2) the representation is not prohibited by law;

11
(3) the representation does not involve the assertion of a claim by one client against another
client represented by the lawyer in the same litigation or other proceeding before a tribunal; and
(4) each affected client gives informed consent, confirmed in writing.
 See also, Restatement § 122(1)
o Model Rule 1.7(a)(1) v. Model Rule 1.9(limited to ―the same or a substantially related matter‖)
c) Corporate clients
– Generally a matter of common sense and fairness, considering actual relationships:
• Divisions: usually treated as part of the same client
• Subsidiaries/Parents: depends on the actual corporate
family context.
• Municipal clients: common sense
– Westinghouse (7th Cir. 1978)
d) Escaping Conflicts
a. If you find that a conflict develops between two clients, may you avoid the problem by ‗‗firing‘‘ one
of the clients and continuing to represent the other? Usually not.
b. The Hot Potato Rule:
–Attorney cannot turn one current disfavored client into a former client
to continue representing the favored client.
•Policy concern: loyalty
–Some flexibility though (Pennwalt Corp.; Gould, Inc.):
•Especially when conflict created through no fault of the attorney‘s (e.g. unrelated business
mergers), courts will sometimes allow firms to drop client instead of disqualifying the firm
completely.
–Court considers actual prejudice and whether confidences and feelings of disloyalty are
actually at risk
a) Advance waivers of future conflicts: Should clients be able to agree at the outset of the representation
that, if a conflict later arises, the lawyer may represent one of the clients against the other?
a. Model Rule 1.7, Comment 22
‗The effectiveness of such waivers is generally determined by the extent to which the client
reasonably understands the material risks that the waiver entails. The more comprehensive the
explanation of the types of future representations that might arise and the actual and reasonably
foreseeable adverse consequences of those representations, the greater the likelihood that the client
will have the requisite understanding. Thus, if the client agrees to consent to a particular type of
conflict with which the client is already familiar, then the consent ordinarily will be effective with
regard to that type of conflict. If the consent is general and open-ended, then the consent ordinarily
will be ineffective, because it is not reasonably likely that the client will have understood the
material risks involved. On the other hand, if the client is an experienced user of the legal services
involved and is reasonably informed regarding the risk that a conflict may arise, such consent is
more likely to be effective, particularly if, e.g., the client is independently represented by other
counsel in giving consent and the consent is limited to future conflicts unrelated to the subject of the
representation.-
 See also Restatement § 122, Comment d:
b) Positional Conflicts
a. Rule 1.7, Comment 24:
 Advocating a legal position that might create precedent adverse to client in an unrelated matter
does not create a conflict of interest.
 •COI if ―there is a significant risk that a lawyer‘s action on behalf of one client will materially
limit the lawyer‘s effectiveness in representing another client in a different case; for example,
when a decision favoring one client will create a precedent likely to seriously weaken the
position taken on behalf of the other client. Factors:
12
- – where the cases are pending,
- – whether the issue is substantive or procedural,
- – the temporal relationship between the matters,
- – the significance of the issue to the immediate and long-term interests of the clients involved
and
- – the clients‘ reasonable expectations in retaining the lawyer.
 If there is significant risk of material limitation, lawyer needs informed consent.

6. Conflicts of Interest in Criminal Litigation; Conflicts of Interest between Client and


Attorney; Insured and Insurer. Casebook, pp.201-228,.229-245
a. Rule 1.7, Comment 23:
 ―conflict may exist by reason of substantial discrepancy in the parties‘ testimony, incompatibility
in positions in relation to an opposing party or the fact that there are substantially different
possibilities of settlement of the claims or liabilities in question…The potential for conflict of
interest in representing multiple defendants in a criminal case is so grave that ordinarily a
lawyer should decline to represent more than one codefendant.‖
 •Court has affirmative obligation to ―promptly inquire with respect to such joint representation
and shall personally advise each defendant of his right to the effective assistance of counsel,
including separate representation.‖ (Federal Rules of Criminal Procedure 44(c))
 •Unless ―good cause to believe no conflict of interest is likely to arise‖…
 •Constitutional (6A) Issue: Ineffective assistance of counsel.
b. Prosecutors requesting separate defense counsel
• Permitted. The Court must inquire if brought to their attention.
• Possible strategic drawbacks:
– clients may benefit from sticking together (avoiding prisoners‘ dilemma); may get
better attorneys; may get free attorneys
– but courts are more concerned with the avoiding conflicts of interest.
– Even where defense objects: US v Locascio (2nd Cir., 1993)
c. Constitutional standards
• Holloway v. Arkansas (1978):
– ―joint representation of conflicting interests is suspect because of what it tends to
prevent the attorney from doing.‖
– For example, wrt to advice about testifying against others in exchange for leniency
• Cuylver v. Sullivan (1980):
– Unless court should have know of COI, def. must prove that there was
an actual COI that adversely affected lawyer‘s performance)
– Strickland (1984): for reversal, def. must show incompetence AND actual
prejudice.
– Burger v. Kemp (1987); Mickens v Taylor (2002): very tough standard.
• This is for overturning convictions. There is a much lower bar for attorney discipline.
d. Representing Clients in Conflict
o Thomas v. State (SC 2001): layer represented husband and wife, prosecutor made offer for them
to each plea to half. Once that happened, any action in either spouse‘s best interest would
unavoidably harm the other. This was not accounted for in the earlier conflict waiver.
o US v. Schwarz (2nd Cir, 2002): Law firm who represented one of Louima‘s torturers also
represented PBA. Nonconsentable confict because lawyer may have been precluded from using
trial tactics that would harm the firms‘ civil
 defense (lawyer without the split loyalties might have blamed other cops).
e. Prosecutor’s Personal Ambitions (PP11)
13
 Of what relevance is it that the prosecutor in this problem is highly motivated to get convictions
that will tend to further his political ambition? Could a prosecutor‟s personal motives ever
constitute a conflict of interest with his or her duties as a public prosecutor?
 Political ambitions are fine, but courts draw the line at actual personal conflicts.
- Greer (Cal., 1977): victim‘s mother worked in prosecutor‘s office and stood to gain
custody from the conviction (also seems to have influenced the arrest);
- Rook (Or., 1976): prosecutor refused to plea bargain because he didn‘t like two attorneys;
- Lewis (Cal., 1997): auditor-controller of OC, alleged misconduct led to cuts in
prosecutor‘s office, Court disqualified entire office.
 Privitization/contingency fees: create risks that prosecutors will be influenced by money (and
less likely to drop cases).
f. Writing about your cases
 Okay if you wait until the conclusion of the trial
Model Rule 1.8(d):
Prior to the conclusion of representation of a client, a lawyer shall not make or negotiate an
agreement giving the lawyer literary/media rights to a portrayal or account based in substantial
part on information relating to the representation.
 Comment 9: An agreement by which a lawyer acquires literary or media rights concerning
the conduct of the representation creates a conflict between the interests of the client and the
personal interests of the lawyer. Measures suitable in the representation of the client may
detract from the publication value of an account of the representation.
g. Stock as Fee Payment
 Fees paid in the form of stock = business transaction (Rule 1.8)
 Rule 1.8(a) requirements
(1) the transaction and terms on which the lawyer acquires the interest are fair and reasonable to
the client and are fully disclosed and transmitted in writing in a manner that can be reasonably
understood by the client;
(2) the client is advised in writing of the desirability of seeking and is given a reasonable
opportunity to seek the advice of independent legal counsel on the transaction; and
(3) the client gives informed consent, in a writing signed by the client, to the essential terms of
the transaction and the lawyer‘s role in the transaction, including whether the lawyer is
representing the client in the transaction.
 Sanctions:
o Professional discipline in bad cases; restitution/voiding of the agreement in others (even
when deal was patently reasonable and in good faith).
 Policy Rationale
 Rule 1.8, Comment 1:
o ―A lawyer‘s legal skill and training, together with the relationship of trust and confidence
between lawyer and client, create the possibility of overreaching when the lawyer
participates in a business, property or financial transaction with a client, for example, a
loan or sales transaction or a lawyer investment on behalf of a client.‖
 •Counter-arguments:
o – Make it easier to afford representation
o – Can help clients in their business operations
o – Can function as a good faith sign
 PP11 - May Doe, without Johnson‘s consent, invest in the parcel near the industrial park? As we
have posed the problem, Johnson does not want to buy the property himself.
 Yes.

14
 Model Rule 1.8(b): a lawyer ‗‗shall not use information relating to representation of a client to
the disadvantage of the client‘‘ unless the client consents.
h. Gifts to Attorneys (Rule 1.8(c))
 May Doe follow Johnson‘s direction to prepare the documents necessary to give herself title to
the Mercedes Benz?
 No. Doe cannot prepare the instrument (1.8(c)).
 Would it be proper for Doe to admire the car and let Johnson know she would like to have it?
 No. A lawyer ―shall not solicit any substantial gift from a client‖ (Rule 1.8(c))
a. Sexual relationships between lawyers and clients
Model Rule 1.8(j):
A lawyer shall not have sexual relations with a client unless a consensual sexual relationship existed
between them when the client lawyer relationship commenced.
- What‘s wrong with sleeping with clients or their significant others?
Rule 1.8, Comment 17:
– Relationship ―almost always unequal‖
– risk of unfair exploitation/ coercion.
– Emotional involvement may cloud lawyer‘s professional judgment
– Confusion about confidentiality of communications
•Long, checkered history of lawyers sleeping with clients, and getting away with it. Courts felt that this
was unseemly but not against the rules.
b. Conflict of Interest?
- • In 2000, a class of 500+ Nextel employees retained Leeds, Morelli & Brown to file employment
discrimination claims against Nextel.
- • With clients‘ consent, Leeds, Morelli & Brown entered into an agreement with Nextel to persuade
the employees to forego litigation and commit to mediation and arbitration. In exchange, Nextel
agreed it would pay the law firm up to $5.5 million, based on the number of claims resolved.
- Should this type of arrangement be allowed?
o Maybe not.
 • The 2nd Circuit said that employees demonstrated sufficient evidence to claim that the law firm
had breached its fiduciary duty. The agreement "created overriding and abiding conflicts of
interest for LMB and thoroughly undermined its ability to deal fairly, honestly, and with
undivided loyalty to appellants," Judge Ralph Winter wrote on behalf of a three-judge panel.
 Conflicting views on whether the insurer is actually a client It‘s a matter of state insurance law: Model
rules offer no direct guidance.
• Majority view seems to be that both (insurer and insured) are clients.
 Rule 1.7a(2) standard applies, either way:
There‘s a conflict if: ―a significant risk that the representation of one or more clients will be materially
limited by the lawyer‘s responsibilities to another client, a former client or a third person or by a
personal interest of the lawyer.‖
 Are the problems of obtaining consent different depending on whether the insurance company is
characterized as a client? (PP12)
 No. Informed consent about fact of payment still required under Model Rule 1.8(f) and any
―materially limiting‖ conflicts under Model Rule 1.7.
 Model Rule 1.8(f)
(f) A lawyer shall not accept compensation for representing a client from one other than the client unless:
(1) the client gives informed consent;
(2) there is no interference with the lawyer‘s independence of professional judgment or with the
client-lawyer relationship; and
(3) information relating to representation of a client is protected as required by Rule 1.6.

15
 Comment 11:
o ―Third-party payers frequently have interests that differ from those of the client, including
interests in minimizing the amount spent on the representation and in learning how the
representation is progressing‖
 occurs in other contexts as well: family members, employers, unions…
 Conflicts of Interest and Confidentiality
 What must a lawyer do when she learns embarrassing or incriminating information about the insured
that the insurance company would want to know?
•Parsons (AZ, 1976):
– attorney owed ―undivided loyalty‖ to insured; not allowed to disclose information to client‘s
detriment to help the insurance company.
– courts often hold that attorney misconduct can waive an insurance carrier‘s defenses against payment.
 Montanez v. Irizarry-Rodriguez (NJ, 1994): attorney could not impeach testimony of insured to
help insurer.
 PP 12 - Would Henderson be barred from representing a client in an unrelated case suing the
insurance company that retains her in other cases to represent its policyholders?
o •Yes.
 – If insurance company is a client, then 1.7(a)(1)
 – If insurance company not a client, 1.7a(2)
 Reservation of Rights
 All–Mutual is defending the insured under a ‗‗reservation of rights‘‘ of the kind in Parsons. Does
that in itself create a conflict of interest for a lawyer?
- •Yes. At this point, separate lawyers are usually required. Lawyer defending the insured must be
sure not to get caught in the middle.
 Reaction to settlement offer? (pp12)
- – Often conflicting interest with respect to settlement offers (insured doesn‘t care if its
- within the limit; insurer might have nothing to gain)
- – In many states, law makes insurer completely liable for ultimate award if they turn down
reasonable settlement.
 What is the lawyer to do if the insurer wants to settle but the insured does not?
- – Insurance law usually gives insurer the right to settle. Insured and attorney generally are not
allowed to thwart this, unless insured wants to sign away his policy.
 Different in professional malpractice context.
- Lawyer must keep client informed though
 In order to reduce the cost of litigation, may the insurer direct the lawyer to take no more than, say,
two depositions in the case?
- Not if it disadvantages the client‘s case (see model rule 1.8(f))
 •Utah advisory opinion- illegal only if it works against the client‘s best interest.
 •Also, several states don‘t allow lawyers to submit their files to insurance companies‘
auditors, for confidentiality reasons.

6. Former Clients. Casebook, pp.245-261


a. Does a lawyer owe the same duty of loyalty to a former client as to a current one?
 Not exactly
b. What standard defines when a lawyer may undertake a matter that is contrary to the interest of a former
client?
 Model Rule 1.9 (Duties to Former Clients)
(a) A lawyer who has formerly represented a client in a matter shall not thereafter represent another
person in the same or a substantially related matter in which that person‘s interests are materially
16
adverse to the interests of the former client unless the former client gives informed consent, confirmed
in writing.
(b) A lawyer shall not knowingly represent a person in the same or a substantially related matter in
which a firm with which the lawyer formerly was associated had previously represented a client
(1) whose interests are materially adverse to that person; and
(2) about whom the lawyer had acquired information protected by Rules 1.6 and 1.9(c) that is
material to the matter; unless the former client gives informed consent, confirmed in writing.
(c) A lawyer who has formerly represented a client in a matter or whose present or former firm has
formerly represented a client in a matter shall not thereafter:
(1) use information relating to the representation to the disadvantage of the former client except
as these Rules would permit or require with respect to a client, or when the information has
become generally known; or
(2) reveal information relating to the representation except as these Rules would permit or
require with respect to a client.
c. Substantially Related
 Should the current medical malpractice case and the former adoption proceeding discussed in this
problem be held to be ‗‗substantially related matters? (PP13)
 Real question is whether they look alike… whether there is overlap.
 Courts apply different tests that generally use similar factors:
 (1) Is it the same case? Are the matters actually related?
 (2) Does it directly involve work the attorney performed for the first client?
 (3) Are the cases factual similar?
 (4) Are there legal similarities?
 (5) the extent of the lawyers‘ involvement in the cases?
 (6) Is there a substantial risk that the lawyer learned confidential information that would help the
client in the second matter?
 Should cases be considered „„substantially related‟‟ if the factual issues in two or more cases are
closely related, but there is no confidentiality?
 Yes, there are often loyalty concerns nonetheless. The Restatement‘s ―accommodation client‖
concept is the minority position.
 Suppose Heath knows no crucial secrets (e.g., that Abraham operates while intoxicated), but she
does have general impressions of Abraham‟s personality and specific knowledge of his financial
situation. Could that make the cases „„substantially related‟‟?
 No. This is generally not enough. Chugach is the minority rule– most courts seem to think it goes
too far.
d. The proof dilemma
Touchstone in most cases is whether lawyer received relevant confidential information in the prior
representation, but how do you prove this without revealing the confidential information that you are
trying to keep secret?
 Courts use an ―objective‖ approach.
Model Rule 1.9, Comment 3:
―A conclusion about the possession of such information may be based on the nature of the services the
lawyer provided the former client and information that would in ordinary practice be learned by a
lawyer providing such services.‖
 Showing of actual prejudice is not required. Rulemakers do not want clients at mercy of their former
attorney‘s ethics in these scenarios.
e. Past Representation of codefendant

17
 Assume that Martha Heath had not represented Dr. Abraham in any earlier malpractice case, but she
had represented a codefendant in such a case. Should that bar her from taking the current case
against Dr. Abraham?
 Quite possibly‐‐It depends on the facts of the past representation.
 If there was cooperation between the defendants then there may be an implied attorney‐client
relationship.
f. Confidential Information
Suppose that, while doing discovery in her earlier representation of Dr. Abraham‟s co‐defendant, Heath
had learned something about a different patient of that doctor—the patient‟s tendency to alcoholism, for
example. Could Heath use that information in a suit against that other patient on behalf of a current
client?
 Only with Dr. Abraham’s consent.
Rule 1.9(c):
a lawyer may not use or reveal information acquired in a prior representation contrary to the interest of
the former client unless the information has become generally known.
 Here Dr. Abraham wouldn‘t consent because of patient‐confidentiality rules, but otherwise it would
be permissible.
g. Past class action clients
Suppose Heath recently settled a class action on behalf of all the residents of her city against a local
department store for systematically overcharging interest on past due accounts. Is Heath now prohibited
from representing the department store suing a customer who happened to be a member of the former class
for failure to pay a department store bill?
 There is generally no obligation of loyalty to past unnamed class action clients.
 BUT
 There may be an obligation of confidentiality, depending on facts of representation and
relationship.
h. Other legal personnel
Suppose that the lawyer does not change firms, but the lawyer‟s nonlawyer assistant, e.g., a secretary or
paralegal, does. Does Model Rule 1.9 have anything to say about that situation? What about law students
(summer associates)?
- No. Model Rule 1.9 only applies to lawyers. Some analogous principles apply via agency law
though.
What about temp attorneys?
- Yes, they are governed by Rule 1.9. It depends on how they were used at their previous firms—
whether they were treated like permanent attorneys (i.e. working on multiple cases, access to files
and the computer network, participated in discussions, etc.)
i. Access to Disqualified counsel‘s work product
If Dr. Abraham‟s motion were granted and Heath were disqualified, should substituted counsel have
access to Heath‟s work product?
 Usually, unless the work product is likely tainted.
 But if counsel is disqualified to protect client confidences, it becomes much harder to prove that
the work product is not also tainted.

5. Imputed Disqualification. Casebook, pp.261-275


a. Model Rule 1.10: Imputation of conflicts of interest
(a)While lawyers are associated in a firm, none of them shall knowingly represent a client when any
one of them practicing alone would be prohibited from doing so by Rules 1.7 or 1.9…
(2) the prohibition is based upon Rule 1.9(a) or (b) and arises out of the disqualified lawyer‘s
association with a prior firm, and
18
(i) the disqualified lawyer is timely screened from any articipation in the matter and is
apportioned no part of the fee therefrom;
(ii) written notice is promptly given to any affected former client to enable the former client to
ascertain compliance with the provisions of this Rule, which shall include a description of the
screening procedures employed; a statement of the firm‘s and of the screened lawyer‘s
compliance with these Rules; a statement that review may be available before a tribunal; and an
agreement by the firm to respond promptly to any written inquiries or objections by the former
client about the screening procedures; and
(iii) certifications of compliance with these Rules and with the screening procedures are provided
to the former client by the screened lawyer and by a partner of the firm, at reasonable intervals
upon the former client‘s written request and upon termination of the screening procedures.

- Should conflicts that are „„personal‟‟ to one lawyer be imputed throughout a firm?
Model Rule 1.10 (a)(1) (exception):
―the prohibition is based on a personal interest of the disqualified lawyer and does not present a
significant risk of materially limiting the representation of the client by the remaining lawyers in
the firm‖
 Personal interests are treated very differently than law firm business conflicts.
• If ―personal conflict‖ is political/philosophical, and not about confidences
or loyalty, then no risk to any client.
• But, if loyalty to the conflicted lawyer might limit representation, then it
becomes a firm conflict.
b. Model Rule 1.9: Duties to Former Clients
(a) A lawyer who has formerly represented a client in a matter shall not thereafter represent another
person in the same or a substantially related matter in which that person‘s interests are materially
adverse to the interests of the former client unless the former client gives informed consent, confirmed
in writing.
c. Disqualification imputed to one attorney does not spread with them when they go to another firm.
d. PP14 - Suppose Xeres resigns from the firm of W & X. Will Willis still be disqualified from
representing World Wide?
 No.
Model Rule 1.10(b)
When a lawyer has terminated an association with a firm, the firm is not prohibited from thereafter
representing a person with interests materially adverse to those of a client represented by the formerly
associated lawyer and not currently represented by the firm, unless:
(1) the matter is the same or substantially related to that in which the formerly associated
lawyer represented the client; and
(2) any lawyer remaining in the firm has information protected by Rules 1.6 and 1.9(c) that is
material to the matter.
(c) A disqualification prescribed by this rule may be waived by the affected client
under the conditions stated in Rule 1.7.
(d) The disqualification of lawyers associated in a firm with former or current
a. government lawyers is governed by Rule 1.11.
e. Pros and Cons of Allowing Screening
 Pros:
- ‐Helps clients secure legal counsel
- ‐Helps lawyers to find new jobs
- ‐Enables firms to merge
- ‐Helps firms make as much money as possible

19
 Cons:
- ‐Client discomfort.
- ‐Appearances of impropriety
 If the court disqualifies the firm of W & X and all of its partners and associates, may C & B simply get
new local counsel? Will the disqualification of W & X require that C & B be disqualified as well?
o Almost certainly.The firms are affiliated for purposes of Comment 1.
–Usually a matter of if they publicly hold themselves out as connected. These facts are not
enough to impute such knowledge to W & X.
f. Miscellaneous
• You can be disqualified if an attorney with whom you share office space is disqualified, if
you are not careful to protect confidences.
• Public Defenders not treated like private law firms:
– but courts still require proof that confidences will be kept
– Wrt some conflicts, court presumes prejudice

6. Advising Clients (Individual and Business Clients). Casebook, pp.291-319, PP 15


a. Model Rule 2.1
• Requires candid advice
• Sometimes even moral advice‐‐ purely technical advice may be inadequate
• Comment 2: lawyer may initiate advice when it appears to be in the client‘s interest
• Comment 4: May require recommendations to consult other professionals
In representing a client, a lawyer shall exercise independent professional judgment and render
candid advice. In rendering advice, a lawyer may refer not only to law but to other considerations
such as moral, economic, social and political factors, that may be relevant to the client‘s situation.
Comment 2:
Advice couched in narrow legal terms may be of little value to a client, especially where practical
considerations, such as cost or effects on other people, are predominant. Purely technical legal
advice,
therefore, can sometimes be inadequate. It is proper for a lawyer to refer to relevant moral and
ethical considerations in giving advice. Although a lawyer is not a moral advisor as such, moral and
ethical
considerations impinge upon most legal questions and may decisively influence how the law will be
applied.
 The S&L cases (FDIC cases) attorneys got in big trouble for accepting their clients‘ representations
at face value, in heavily regulated industries where catastrophic consequences possible.
b. Giving Advice re: crime and fraud
• No exception here– the rule is clear. Lawyer may not counsel or assist a client to commit a
crime or fraud (Rule 1.2)
– Does not mean that you cannot answer any questions about legally suspect
actions.
– Cannot tell them how to commit a crime with impunity
• Does this really seem straightforward though? Any ambiguities?
– What does it really mean to ―counsel‖ or ―assist‖?
– What if your client wants to know the likelihood of being punished for speeding a
certain amount?
– What if they say that they‘re just curious?
– Should it matter if it‘s a serious crime or not?
– What if the lawyer expressly states that he is not advising them to commit a crime or
tells them not to?
20
• Under ABA rules, you are not allowed to advise a group who you are certain will regularly
violate the law, like an organized crime family.
c. Model Rule 1.2(d)
(d) A lawyer shall not counsel a client to engage, or assist a client, in conduct that the lawyer knows
is criminal or fraudulent, but a lawyer may discuss the legal consequences of any proposed course of
conduct with a client and may counsel or assist a client to make a good faith effort to determine the
validity, scope, meaning or application of the law.
Comment 9:
―…This prohibition, however, does not preclude the lawyer from giving an honest opinion about the
actual consequences that appear likely to result from a client‘s conduct. Nor does the fact that a
client uses advice in a course of action that is criminal or fraudulent of itself make a lawyer a party
to the course of action. There is a critical distinction between presenting an analysis of legal aspects
of question‐ able conduct and recommending the means by which a crime or fraud might be
committed with impunity.
d. Rule 1.13(a)
A lawyer employed or retained by an organization represents the organization acting through its duly
authorized constituents.
• The premise of Rule 1.13 is to make sure that the clients‘ representatives are acting in
accordance with the client‘s true interests.
• The exhaustion of internal remedies is a way to ensure you really know what the corporation
wants.
e. Rule 1.13(b)
If a lawyer for an organization knows that an officer, employee or other person associated with the
organization is engaged in action, intends to act or refuses to act in a matter related to the
representation that is a violation of a legal obligation to the organization, or a violation of law that
reasonably might be imputed to the organization, and that is likely to result in substantial injury to
the organization, then the lawyer shall proceed as is reasonably necessary in the best interest of
the organization. Unless the lawyer reasonably believes that it is not necessary in the best interest
of the organization to do so, the lawyer shall refer the matter to higher authority in the
organization, including, if warranted by the circumstances, to the highest authority that can act on
behalf of the organization as determined by applicable law.
f. Fired?
 Being fired does not necessarily relieve you of your ethical responsibilities.
Rule 1.13(e)
 ―A lawyer who reasonably believes that he or she has been discharged because of the lawyer‘s
actions taken pursuant to paragraphs (b) or (c), or who withdraws under circumstances that
require or permit the lawyer to take action under either of those paragraphs, shall proceed as the
lawyer reasonably believes necessary to assure that the organization‘s highest authority is
informed of the lawyer‘s discharge or withdrawal.‖
 • You would probably need to inform the Board of Directors (in writing)
 • Lawyer may be liable to the client, if they fail to adequately investigate or inform other
people.
g. May you report the corporate client to a public enforcement agency?
 Rule 1.13(c) and Comment 6
(c) Except as provided in paragraph (d), if
(1) despite the lawyer‘s efforts in accordance with paragraph (b) the highest authority that can
act on behalf of the organization insists upon or fails to address in a timely and appropriate
manner an action or a refusal to act, that is clearly a violation of law, and
(2) the lawyer reasonably believes that the violation is reasonably certain to result in
substantial injury to the organization then the lawyer may reveal information relating to the
21
representation whether or not Rule 1.6 permits such disclosure, but only if and to the extent the
lawyer reasonably believes necessary to prevent substantial injury to the organization.
• Failure to inform will not render lawyer automatically liable, but it seems possible (under Tarasoff
logic) and basic tort law.
h. Attorneys serving as directors
If you are Sleepware‘s counsel, can you also serve as a director?
• Common practice, clearly there is no per se prohibition. Rule 1.7(b), Comment 35
– a bunch of potential conflicts that you should consider
though.
• Lawyer as witness
• Lawyer with personal interests in tension with the corporation‘s
• Lawyer not being objective because of personal involvement
• Ambiguity over A‐C privilege
– But benefits: prestige and money for attorney; good way to keep clients in some instances.

5. Contact with Represented and Unrepresented Persons. Casebook, pp.320-336


a. Model Rule 4.2:
In representing a client, a lawyer shall not communicate about the subject of the representation with
a person the lawyer knows to be represented by another lawyer in the matter, unless the lawyer has
theconsent of the other lawyer or is authorized to do so by law or a court order.
 Comment 7:
In the case of a represented organization, this Rule prohibits communications with a
constituent of the organization who supervises, directs or regularly consults with the
organization’s lawyer concerning the matter or has authority to obligate the organization
with respect to the matter or whose act or omission in connection with the matter may be
imputed to the organization for purposes of civil or criminal liability. Consent of the
organization‘s lawyer is not required for communication with a former constituent. If a
constituent of the organization is represented in the matter by his or her own counsel, the consent
by that counsel to a communication will be sufficient for purposes of this Rule.
 Compare Rule 3.4(f). In communicating with a current or former constituent of an
organization, a lawyer must not use methods of obtaining evidence that violate the legal
rights of the organization. See Rule 4.4.
 Pros:
- Protects clients from overreaching from opposing counsel and from
- inadvertently saying things against their interests; prevents lawyers from
- stealing others‘ clients.
- ―contributes to the proper functioning of the legal system by protecting a person who
- has chosen to be represented by a lawyer in a matter against possible overreaching by
- other lawyers who are participating in the matter, interference by those lawyers with
- the client‐lawyer relationship and the uncounselled disclosure of information relating
- to the representation.‖ [Comment 1]
 Cons:
- Can frustrate clients‘ wishes, increase legal costs, slow the resolution of conflicts.
b. Direct communication between parties
 • Rule does not prevent this.
- – Lawyers can give advice but cannot mastermind the entire encounter.
- – Unclear where/how you draw the line though.
c. PP 16 - Can Shabazz interview Barry Winters, Mary Speedy, and the truck driver?
Yes, no, and no.

22
Straightforward application of Comment 7:
•Mary Speedy regularly consults with counsel and has authority to obligate the company.
•The truck driver‘s act may be imputed to the organization
•Only Barry Winters is fair game to interview in absence of counsel.
d. Non‐represented persons
What limits apply to his contact with Winters?
Model Rules 4.1, 4.3, and 4.4 are all still relevant
• 4.1: lawyer may not lie
• 4.3 limits what lawyer can say to unrepresented people
o Cannot make them think you are disinterested or on their side(see Comment 2)
• 4.4 lawyer cannot harass people
e. Lawyer as party
There is disagreement about whether the rule applies if the lawyer is a party.
• ABA rules and DC opinion: lawyer is a lawyer for all purposes; same rules apply.
• Restatement disagrees.
a. – There are questions of fairness and public policy.
f. What rule governs how Bentley should handle her interviews with her the truck driver?
Rules 4.3
In dealing on behalf of a client with a person who is not represented by counsel, a lawyer shall not
state or imply that the lawyer is disinterested. When the lawyer knows or reasonably should know
that the unrepresented person misunderstands the lawyer‘s role in the matter, the lawyer shall make
reasonable efforts to correct the misunderstanding. The lawyer shall not give legal advice to an
unrepresented person, other than the advice to secure counsel, if the lawyer knows or reasonably
should know that the interests of such a person are or have a reasonable possibility of being in
conflict with the interests of the client.
o [Comment 1] An unrepresented person, particularly one not experienced in dealing with legal
matters, might assume that a lawyer is disinterested in loyalties or is adisinterested authority on
the law even when the lawyer represents a client. In order to avoid a misunderstanding, a lawyer
will typically need to identify the lawyer‘s client and, where necessary, explain that the client has
interests opposed to those of the unrepresented person. For misunderstandings that sometimes
arise when a lawyer for an organization deals with an unrepresented constituent, see Rule
1.13(f).
o [Comment 2] The Rule distinguishes between situations involving unrepresented persons whose
interests may be adverse to those of the lawyer‘s client and those in which the person‘s interests are
not in conflict with the client‘s. In the former situation, the possibility that the lawyer will
compromise the unrepresented person‘s interests is so great that the Rule prohibits the giving of any
advice, apart from the advice to obtain counsel. Whether a lawyer is giving impermissible advice
may depend on the experience and sophistication of the unrepresented person, as well as the setting
in which the behavior and comments occur.
o Note: Some courts require more.
g. Should Bentley announce that she is the attorney for all employees personally, thus making all of
them ‗represented persons‘ under Rule 4.2? No.
- • Conflicts of interest, she needs their consent
h. Former Employees are generally fair game
 Unless they are represented by independent counsel
i. May Shabazz, or an investigator on his behalf, phone a receptionist at Speedy Corp., and ask the
receptionist if Speedy Corp. sanctions its drivers who are late? What if any prospective customer can
do that?
 Probably ok. Depends on whether this is considered to be ―the subject of the representation.‖
This seems general enough not to fall into that category.
23
6. Ethics of Negotiation Casebook, pp.336-352
a. PP17, prob 20 When you discuss the case with the prosecutor, can you assert that Young had nothing
alcoholic to drink the day of the accident?
a. No. You can make statements about how they‘ll never be able to prove it, maybe.
b. Should there be higher obligations of truthfulness in negotiations? Do lawyers have a higher obligation
to be truthful than non‐lawyers? [Burden really not all that great on attorneys here. 4.1 only ―false
statement of material fact or law‖
c. What is the first potential ethical issue here?
Lawyer has not consulted with the client about objectives of representation.
Model Rule 1.2 (a)
…a lawyer shall abide by a client‘s decisions concerning the objectives of representation and, as
required by Rule 1.4, shall consult with the client as to the means by which they are to be pursued. A
lawyer may take such action on behalf of the client as is impliedly authorized to carry out the
representation. A lawyer shall abide by a client‘s decision whether to settle a matter...
Model Rule 1.4
(a) A lawyer shall:
(1) promptly inform the client of any decision or circumstance with respect to which the client‘s
informed consent…;
(2) reasonably consult with the client about the means by which the client‘s objectives are to be
accomplished;
(3) keep the client reasonably informed about the status of the matter;
(4) promptly comply with reasonable requests for information…
(b) A lawyer shall explain a matter to the extent reasonably necessary to permit the client to make
informed decisions regarding the representation.
d. What rule explains which types of statements are impermissible during negotiations?
Model Rule 4.1
In the course of representing a client a lawyer shall not knowingly:
(a) make a false statement of material fact or law to a third person; or
(b) fail to disclose a material fact when disclosure is necessary to avoid assisting a criminal or fraudulent
act by a client, unless disclosure is prohibited by Rule 1.6.
o Lower standard than what would be required in court.
- The principle behind this isn‘t clear. Seems to just be that we all know people exaggerate and bluff
during negotiations so you would be a fool to rely on your opponent too much.
- Comment 1
A lawyer is required to be truthful when dealing with others on a client‘s behalf, but generally has no
affirmative duty to inform an opposing party of relevant facts. A misrepresentation can occur if the
lawyer incorporates or affirms a statement of another person that the lawyer knows is false.
Misrepresentations can also occur by partially true but misleading statements or omissions that are the
equivalent of affirmative false statements. For dishonest conduct that does not amount to a false
statement or for misrepresentations by a lawyer other than in the course of representing a client, see Rule
8.4.
Comment 2
This Rule refers to statements of fact. Whether a particular statement should be regarded as one of fact
can depend on the circumstances. Under generally accepted conventions in negotiation, certain types of
statements ordinarily are not taken as statements of material fact. Estimates of price or value placed on
the subject of a transaction and a party‘s intentions as to an acceptable settlement of a claim are
ordinarily in this category…Lawyers should be mindful of their obligations under applicable law to
avoid criminal and tortuous misrepresentation.

24
- Not a license to commit fraud.
e. Higher standard for prosecutors
Model Rule 3.8
The prosecutor in a criminal case shall:
….
(d) make timely disclosure to the defense of all evidence or information known to the prosecutor that
tends to negate the guilt of the accused or mitigates the offense, and, in connection with sentencing,
disclose to the defense and to the tribunal all unprivileged mitigating information known to the
prosecutor, except when the prosecutor is relieved of this responsibility by a protective order of the
tribunal;
- Somewhat broader than Brady, which requires that defense request evidence.
Model Rule 3.4
A lawyer shall not:
….
(e) in trial, allude to any matter that the lawyer does not reasonably believe is relevant or that will not be
supported by admissible evidence, assert personal knowledge of facts in issue except when testifying as
a witness, or state a personal opinion as to the justness of a cause, the credibility of a witness, the
culpability of a civil litigant or the guilt or innocence of an accused
f. Would the lawyer have a lesser duty of disclosure when making representations on behalf of a financial
institution client in the course of its review by a federal agency?
 Of course not. Under Rule 4.1, lawyer may even have to affirmatively disclose certain
adverse facts if client is obligated to do so.
g. Should there be any limit on what parties to a negotiation may agree? May a lawyer agree, for example,
that in exchange for a generous financial payment to the lawyer‟s client, neither the lawyer nor the
client will file criminal charges against the opposing party (assuming that the party engaged in conduct
that would justify such charges)?
 Modern rule seems to be that you can do so, as long as it doesn‘t amount to extortion under
state law.
 ABA Formal Opinion 92‐363: Requires that both civil and criminal case are well founded in
fact and law.
 ABA Formal Opinion 94‐383: Might not be ok to threaten the other attorneys though
h. You have discovered information that the city would prefer be kept quiet. Assume that the city proposes
to drop all charges against Young in exchange for your agreement to never defend another drunk driving
case. Do the Model Rules permit you to make such a deal for Young?
No. Model Rule 5.6(b)
A lawyer shall not participate in offering or making:
….
(b) an agreement in which a restriction on the lawyer‘s right to practice is part of the settlement
of a client controversy.
Is this a sound rule?
But what if defendant– after the case is over– wants to hire the plaintiff‘s attorney as a
consultant?
 Does not violate any rule.
i. (PP17) Suppose that in investigating Young‘s case you learn that his car had a serious steering defect
that could have involved him in an even more serious accident. You realize that thousands of other
drivers have cars with the same defect that the manufacturer has not acknowledged. May you negotiate a
handsome settlement for Young that includes a provision that you will keep what you know confidential
and not use it on behalf of any future client?
 Probably.

25
 Obvious public policy reasons to oppose this practice, but does not violate any specific rule.
j. In this Problem, the insurance company has offered to pay $20,000 to settle a total of $25,000 in claims.
May counsel negotiate a settlement that would pay the other driver $18,000 for his personal injuries and
then tell his other client, the car owner, that there is only $2,000 left to pay for the property damage?
 Seems possible, but very problematic.
k. Rule 1.8(g) and Comment 13
(g) A lawyer who represents two or more clients shall not participate in making an aggregate settlement
of the claims of or against the clients, or in a criminal case an aggregated agreement as to guilty or nolo
contendere pleas, unless each client gives informed consent, in a writing signed by the client. The
lawyer‘s disclosure shall include the existence and nature of all the claims or pleas involved and of the
participation of each person in the settlement.
 Joint representation and aggregate settlements raise special problems:
•Risk of differences in willingness to make or accept an offer of settlement
•Each client has the right to have the final say in deciding offer of settlement or plea offer (under
1.2)

5. Lawyer as Evaluator. Casebook, pp. 353-366


l. Model Rule 2.3, Comment 1:
[1] An evaluation may be performed at the client's direction or when impliedly authorized in order to
carry out the representation. See Rule 1.2. Such an evaluation may be for the primary purpose of
establishing information for the benefit of third parties; for example, an opinion concerning the title of
property rendered at the behest of a vendor for the information of a prospective purchaser, or at the
behest of a borrower for the information of a prospective lender. In some situations, the evaluation may
be required by a government agency; for example, an opinion concerning the legality of the securities
registered for sale under the securities laws. In other instances, the evaluation may be required by a third
person, such as a purchaser of a business.
Comment 4:
 ―In no circumstances is the lawyer permitted to knowingly make a false statement of material fact or
law in providing an evaluation under this rule.‖ (see also Model Rule 1.2(d); 4.1)
 Lawyer should have whatever latitude of investigation seems necessary as a matter of professional
judgment.
 Under some circumstances, however, the terms of the evaluation may be limited.
o Certain issues or sources categorically excluded
o Scope of search may be limited by time constraints or the noncooperation of persons having
relevant information.
 Any such limitations that are material to the evaluation should be described in the report.
m. Model Rule 2.1
In representing a client, a lawyer shall exercise independent professional judgment and render candid
advice.‖ (see also Comment 1)
 Sometimes attorney must refuse to write the opinion:
o If the facts appear to be untrue, inconsistent or suspicious
o If the investigation has been so limited that the facts are not reliable
 You may not let a client purchase your ratification of its officials‘ socially beneficial objectives
in the guise of a legal opinion. Model 2.1 and comment 1(rule 2.3) still apply.
 PP18, slide 9: there are specific limits for tax avoidance issues
o ABA formal opinion 85-352: ―lawyer can only take position if there is some realistic
possibility of success if the matter is litigated‖
o Probability of success should approach 1/3
o Not okay to argue unconstitutionality of tax code
26
n. Lawyer must ask the client for consent to give the opinion to the bank even the client as already asked
the lawyer to prepare it.
a. Rule 2.3 (b)
 When the lawyer knows or reasonably should know that the evaluation is likely to affect the
client‘s interests materially and adversely, the lawyer shall not provide the evaluation unless the
client gives informed consent.
o. Reports or opinions arising out of internal affairs may or may not be privileged.
a. It depends if the information is to be disclosed to someone other than the client.
 If truly internal, for the sake of legal advice, then privileged.
 If going to be handed over to another party (bank, for ex.) then no privilege.
 Facts that are discovered but not require to be disclosed remain privileged.
p. If you find damaging facts during the court of investigating the company to issue an opinion:
a. Try to help client re-negotiate the scope of the opinion requested
b. Try to convince your client to remove itself from the situation requiring the opinion
c. Cannot just hide or ignore responsive, damaging information if it would render your opinion
inaccurate or false.
q. Model Rule 2.3, comment 6 – responsibilities in replying to independent auditors (pp18, p13)
a. When a question concerning the legal situation of a client arises at the instance of the client‘s
financial auditor and the question is referred to the lawyer, the lawyer‘s response may be made in
accordance with procedures recognized in the legal profession.
b. ABA/AICA Treaty
 Unasserted claims clause, lawyer should furnish information ―only if the client has determined
that it is probable that a possible claim will be asserted, that there is a reasonable possibility that
the outcome will be unfavorable, and that the resulting liability would be material to the
financial condition of the client.‖
r. Will you be liable to investors if your advice about the likely success of a tax shelter turns out to be
erroneous?
a. Quite possibly
 IRS may sanction attorney too, especially if you express more than 50% chance that the IRA will
accept tax shelter (reliance opinion)

6. Client Fraud and SEC Issues. Casebook, pp.367-384


a. Privilege and Confidentiality
a. What is the legal status of what you have learned from the director of research and development? Is
the information privileged and/or confidential?
 • Probably A‐C privilege, but btw attorney and corporation.
– Director is a high‐ranking official of the corporate client, making confidential
communication, presumably for purpose of legal advice… (not just gossip)
b. 1.6 confidentiality?
a. • Yes.
 – Presumably ―information relating to the representation,‖ broadly construed.
c. Disclosure
May you disclose the possible fraud outside the corporation? Which rules might allow this type of
disclosure?
 Rule 1.13(c)
o allows disclosure outside the corporate client after ―climbing the ladder‖ if:
(1) despite the lawyer‘s efforts…the highest authority that can act on behalf of the organization
insists upon or fails to address in a timely and appropriate manner an action or a refusal to act, that is
clearly a violation of law, and

27
(2) the lawyer reasonably believes that the violation is reasonably certain to result in substantial
injury to the organization, then the lawyer may reveal information relating to the representation
whether or not Rule 1.6 permits such disclosure, but only if and to the extent the lawyer reasonably
believes necessary to prevent substantial injury to the organization.
 Model Rule 1.6(b)
o allows disclosure if lawyer reasonably believes necessary:
(2) to prevent the client from committing a crime or fraud that is reasonably certain to result in
substantial injury to the financial interests or property of another and in furtherance of which the
client has used or is using the lawyer‘s services;
(3) to prevent, mitigate or rectify substantial injury to the financial interests or property of another
that is reasonably certain to result or has resulted from the client‘s commission of a crime or fraud
in furtherance of which the client has used the lawyer‘s services
 Any possible obligation to report?
• Model Rule 1.2(d) forbids attorneys from assisting in clients fraud
• Model Rule 1.13(b) requires the lawyer to ―proceed as is reasonably necessary in the best interest or
the organization‖
• Model Rule 4.1(b) requires lawyer to disclose information necessary to avoid knowingly assisting a
client to commit a crime or fraud
d. Notice of withdrawal
If you do withdraw, should you give a “notice of withdrawal” to other parties?
 ABA Formal Opinion 92‐366:
A lawyer who knows or with reason believes that her services or work product are being used or are
intended to be used by a client to perpetrate a fraud must withdraw from further representation of the
client, and may disaffirm documents prepared in the course of the representation that are being, or will
be, used in furtherance of the fraud, even though such a ―noisy‖ withdrawal may have the collateral
effect of inferentially revealing client confidences.
 Typical language something like: ―I have withdrawn for ethical/professional reasons. Do not rely
on my participation in this matter.‖
 Model Rule 1.2, Comment 10: ―In some cases, withdrawal alone might be insufficient.
It may be necessary for the lawyer to give notice of the fact of withdrawal and to
disaffirm any opinion, document, affirmation or the like. See Rule 4.1.‖
 Controversial. Notice of withdrawal is a compromise that doesn‘t really please either side. Why?
e. Advocacy in Nonadjudicative Proceedings
May you argue that the Dept. of Energy should not test your client‟s device when you know the device
would not pass the test?
Probably not.
 Model Rules 4.1-4.4 govern
 Rule 3.9, comment 3 excludes representation of a client in a negotiation or other
bilateral transaction with a governmental agency or in connection with an application
for a license or other privilege, etc.
 Under 4.1 (and 1.2), lawyer is forbidden from furthering a course of fraudulent
conduct. That would likely be the case here.
f. SEC regulations standard
Do the SEC regulations require us to report the director‟s claims? What is the standard?
17 CFR §§205.2(e) requires evidence of a material violation:
―credible evidence, based upon which it would be unreasonable, under the circumstances, for a prudent
and competent attorney not to conclude that it is reasonably likely that a material violation has
occurred, is ongoing, or is about to occur.‖
 Material violation: any material violation of state or federal securities law, material breach of a fiduciary
duty, or violation of any federal or state law.
28
o Do we have that here? Violation would be material, but the evidence is not there yet.
g. Lawyers’ duties to investigate
May the securities lawyer accept the client‟s representations about matters to be disclosed? Must the lawyer
independently investigate the client?
 Under ABA formal opinions and case law, lawyer usually permitted to accept clients‘
information (see ABA Formal opinion 335).
BUT
Lawyer must investigate if anything seems suspicious, incomplete, or incorrect. Willful
ignorance is not allowed. If lawyer should know that further investigation would
prove that statements are false, then they are false (see ABA Formal Opinion 346).
Does this apply here? Probably. Information from friend/director should raise suspicions about
truthfulness of legal opinion.
h. SEC Authority: Policy considerations?
Should the SEC have the authority to discipline attorneys whose conduct results in inadequate
disclosures or other violations of securities laws?
 Pros:
‐substantive expertise
‐additional oversight
 Cons:
‐May try to unduly harass or coerce attorneys
‐More appropriate for the Bar to discipline attorneys
‐Slippery slope (other govt. agencies may be next)
o Why should the desire to protect the public against securities fraud require a compromise of lawyers‟
ethical standards?
o Lawyer is preparing documents that he knows (potentially many) 3rd parties will rely upon in
making their investment decisions.
o Not like dealings with IRS where you are just giving advice to client and the IRS will not depend
on lawyers‘ statements/advice.
o However, in practice it doesn‘t necessarily make a big difference. Securities fraud is ordinarily a
continuing crime, so there would be no obligation to protect those confidences anyway.
o How do the requirements of Model Rule 1.13 and the SEC regulations differ?
 • SEC regs apply only to people ―appearing and practicing‖ before the SEC.
 • 1.13 limited to information ―related to the representation‖
 • 1.13 specifies ―violation of a legal obligation to the organization‖ or legal violation that might
be imputed to the organization, that is ―likely to result in substantial injury to the organization.‖
 • 1.13 requires actual knowledge; SEC regulations require awareness of evidence such that it
would be unreasonable for an attorney not to conclude that fraud is likely afoot.
o Under SEC regulations…If lawyer works under a supervising attorney, can just report to them.
 Otherwise:
 • Lawyer must: report the problem to the client‘s chief legal officer (CLO).
 • CLO must investigate the problem and report back.
 • Unless the reporting lawyer receives a timely and appropriate response, she must report
the problem to the client‘s audit or compliance committee or to the client‘s board of
directors.
 • If she does not get a satisfactory response, she must explain that to them.
 • Under SEC regulations, permitted to disclose company‘s conduct to the SEC ―to the
extent the attorney reasonably believes necessary‖ to prevent or rectify conduct ―likely to
cause substantial injury to the financial interest of property of the the issuer or investors.‖
(adapted before the new 1.6 and 1.13, which allow permissive disclosure)

29
 • This does not apply if lawyer has been hired to investigate wrongdoing that has already
been reported.
i. Policy concerns
a. Should you be liable to investors in an offering of securities if your investigation is inadequate or if
you fail to prevent fraudulent conduct by your clients?
 Courts have held that the securities statutes don‘t provide private cause of action for civil
damages based on ―aiding and abetting‖ claims. (see Central Bank andStoneridge)
 • But under case law (Mehaffy, Enron), law firms can still be liable for their own negligent,
material misstatements.

5. Ethical Issues in Litigation. Casebook, pp.385-403


a. Model Rule 3.1
A lawyer shall not bring or defend a proceeding, or assert or controvert an issue therein, unless there is a
basis in law and fact for doing so that is not frivolous, which includes a good faith argument for an
extension, modification or reversal of existing law. A lawyer for the defendant in a criminal proceeding,
or the respondent in a proceeding that could result in incarceration, may nevertheless so defend the
proceeding as to require that every element of the case be established
 ―not frivolous‖ standard.
 ‐Objective standard, no reference to lawyer‘s state of mind.
 ABA position is that client‘s purpose is not relevant to the objective merits of the client‘s claim
• Thus, mixed motives are permissible.
Comment 2:
The filing of an action or defense or similar action taken for a client is not frivolous merely because the
facts have not first been fully substantiated or because the lawyer expects to develop vital evidence only
by discovery. What is required of lawyers, however, is that they inform themselves about the facts of
their clients‘ cases and the applicable law and determine that they can make good faith arguments in
support of their clients‘ positions. Such action is not frivolous even though the lawyer believes that the
client‘s position ultimately will not prevail. The action is frivolous, however, if the lawyer is unable
either to make a good faith argument on the merits of the action taken or to support the action taken by a
good faith argument for an extension, modification or reversal of existing law.
Model Rule 3.1 case law
• Fairly high bar
• These cases tend to involve lawyers making truly baseless, groundless claims
– Sometimes bad motive is evident (e.g. ridiculous defamation and discrimination suits)
– Zdravkovich (Md. 2000) (lawyer violated 3.1 by attempting to remove action to federal
court when clearly not allowed)
– In re Richards (N.M. 1999) (lawyer relied on clearly inapplicable precedent)
– Filing a time‐barred case is not necessarily frivolous Model Rule 3.1 case law
Model Rule 3.1 in the criminal defense context
Does it matter whether the appeal is a criminal conviction or civil verdict?
Model Rule 3.1, Comment 3:
―The lawyer‘s obligations under this Rule are subordinate to federal or state constitutional law that
entitles a defendant in a criminal matter to the assistance of counsel in presenting a claim or contention
that otherwise would be prohibited by this Rule.‖
 Lawyers can enter non‐guilty pleas and defend their clients even if the plea is frivolous. Lawyer
can make the prosecutor meet its constitutional burden of proving every element of the offense.
o DC Ethics Opinion 320 (2003): Arguments for jury nullification of existing law are also
permissible. But Tenn. Ethics Opinion 88‐F‐117 (1988): attorneys cannot file motions to
suppress with no basis

30
 This is not a right (or requirement) to raise frivolous issues at trial or on appeal, or even a
requirement to raise every non‐frivolous issue
b. Other Rules Against Frivolous Litigation: Federal Rules of Civil Procedure, Rule 11
(a) Paper shall be signed by attorney of record in the attorney‘s individual name…
(b) Signee certifies that to the best of the person‘s knowledge, information, and belief, formed after
an inquiry reasonable under the circumstances,—
 • No improper purpose (harassment, unnecessary delay or needless increase in litigation costs)
 • The legal claims, have merit (either warranted by existing law or by a nonfrivolous argument
for the extension, modification, or reversal)
 • All factual contentions (or denials) have evidentiary support or are likely to after further
investigation or discovery
 Applies to all papers submitted to trial courts in civil actions (excluding discovery materials).
 • It is implied that the lawyer should read the document.
 • Each claim is analyzed separately.
 • Other improper purposes: publicity, embarrassment or intimidation, or to gain inappropriate
leverage
 • Courts have different positions about the mixed motives issue:
o – Some courts (2nd Circuit and 4th Circuit) find that bad motives do not matter if there is
also a valid purpose and warranted judicial relief.
o – Other courts (5th Circuit)– this does not apply if it can be established that improper motive
was a ―but for‖ cause of the filing.
o – Non‐frivolous initial filings almost always proper, but non‐frivolous motions may be found
to violate Rule 11 if clearly driven by improper motives.
 Frivolous legal claims under Rule 11
o • Legally frivolous if ―no chance‖ of success under existing law.
o • Can be difficult to determine whether arguments to change the law are frivolous or not
 – Litigants are not generally forbidden from requesting that court reconsider past rulings
or interpretations
1. • Candor before the tribunal is advisable here.
2. • Depends on the nature of the claim: generally must be some basis to believe that
Court may rule differently than in past.
o – Objective standard: ―empty head, pure heart‖ not a defense.
o – Following client‘s wishes/commands no defense.
 Possible Rule 11 Sanctions
o • Sanctions up to the trial court‘s discretion, but are limited to deterrence considerations.
 Monetary
1. – Penalties paid to the court
a. • Modest sanctions usually appropriate but depends on the nature of the offense
2. – Attorney‘s fees (only relating to the Rule 11 violation)
 • Non‐monetary:
1. – striking the offending paper;
2. – reprimand;
3. – requiring offender to attend seminars
o NOTE: sanctions intended to deter the attorney from violations, not the client.
o Not clear that the sanctions would be high in our case, if there is no pattern of such conduct,
no intent to injure. (pp20, slide 19)
 Policy trade‐offs?
o Pros:
 ‐Forces lawyers to do homework; reduces costs to courts and opposing parties

31
o Cons:
o ‐Can lead to drawn out collateral litigation
 ‐Sometimes Rule 11 motions over other side‘s Rule 11 motions
o ‐Can be used to bully attorneys
o ‐Can be unpredictable and inconsistently applied, since judges have a lot of discretion
c. Attorney Obligations
 • Lawyer must have evidentiary support for factual allegations (includes client‘s first‐hand
version of events)
 • If no first‐hand evidence, lawyer must identify those items as likely having evidence that will
be found during discovery.
 • Failing to make adequate verification of the facts may result in sanctions (including
opponents‘ attorneys‘ fees)
 • Lawyers‘ investigation burden depends on the circumstances:
o – When time is of the essence, attorney may have less burden, but must follow up promptly.
d. Requirement to expedite litigation
Model Rule 3.2:
―A lawyer shall make reasonable efforts to expedite litigation consistent with the interests of the client.‖
e. •Benefits of improper delay do not count as an interest of the client.
f. •―Nor will a failure to expedite be reasonable if done for the purpose of frustrating an opposing
party‘s attempt to obtain rightful redress or repose…The question is whether a competent lawyer
acting in good faith would regard the course of action as having some substantial purpose other
than delay. Realizing financial or other benefit from otherwise improper delay in litigation is not
a legitimate interest of the client.‖
 Mixed motives still permissible
e. Obligations to make witness available
What obligation do you have to make witnesses available for trial and for deposition of the opposing
party? Can lawyer suggest non‐emergency surgery?
Model Rule 3.4(a)
Lawyer shall not ―unlawfully obstruct another party‘s access to evidence or unlawfully alter, destroy or
conceal a document or other material having potential evidentiary value. A lawyer shall not counsel or
assist another person to do any such act‖
Is that really what is going on here? Here, suggestion would not be ―unlawfully obstructing‖ per se.
Still, lawyer may be sanctioned for bad faith under Chevron Chemical reasoning. Maybe broad reading
of 8.4(d): ―conduct that is prejudicial to the administration of Justice‖
a. ―Third‐party neutrals‖
Model Rule 3.4
• Third‐party neutral assists two or more persons who are not clients of the lawyer to reach a resolution of
a dispute or other matter that has arisen between them.
• May include service as an arbitrator or mediator.
• Lawyer serving as a third‐party neutral shall inform unrepresented parties that the lawyer is not
representing
them and, when necessary, should explain the difference.
– Extra requirement because unlike non‐lawyers who serve as third‐party neutrals, there is a
significant potential for confusion when the parties are unrepresented in the process (Comment 3).

6. Litigation Tactics. Casebook, pp.403-421


a. Why care?
Pros:
‐Pursuit of truth/ administration of justice
32
‐Notions of fair play in an adversarial system
‐Respect for witnesses, parties, etc.
‐Preserving public image and confidence
Cons:
‐May have chilling effect on creative, zealous representation
‐Lack or rules leads to unevenly applied sanctions
‐Possible 1st Amendment issues too
b. Wearing different clothes to court as you do the office (PP 21, p6)
- Should Martin be subject to criticism for wearing different clothes in court than he wears to the
office, for example? What rule(s) might apply here?
 Probably none.
- • State v. Cherryhomes (N.M, 1992) (fined for wearing a bandanna instead of a tie)
c. RULE 8.4: MISCONDUCT
It is professional misconduct for a lawyer to:…
(c) engage in conduct involving dishonesty, fraud, deceit or misrepresentation;
(d) engage in conduct that is prejudicial to the administration of justice…
Model Rule 8.4, Comment 3:
―A lawyer who, in the course of representing a client, knowingly manifests by words or conduct, bias or
prejudice based upon race, sex, religion, national origin, disability, age, sexual orientation or
socioeconomic status, violates paragraph (d) when such actions are prejudicial to the administration of
justice. Legitimate advocacy respecting the foregoing factors does not violate paragraph (d). A trial
judge‘s finding that peremptory challenges were exercised on a discriminatory basis does not alone
establish a violation of this rule.‖
• Georgia v. McCollum (1992): unconstitutional for defense to exercise peremptory challenges
based on race.
• J.E.B. v. Alabama ex rel. T.B. (1994) (sex‐based challenges)
• Leesville Concrete (1991) (civil litigation)
• United States v. Omoruyi (9th Cir., 1993) (unmarried females = sex discrimination)
 Virgin Islands v. Weatherwax (3rd. Cir, 1996): court found lawyer‘s racial decision
permissible
d. Can a lawyer aggressively question witnesses whom the lawyer believes is telling the truth?
Model Rule 4.4(a)
• (a) In representing a client, a lawyer shall not use means that have no substantial purpose other than to
embarrass, delay, or burden a third person, or use methods of obtaining evidence that violate the legal
rights of such a person.
• What is the primary concern?
 In re Zawada (AZ, 2004): Prosecutor accused expert witnesses of fabricating testimony (with no
evidence). Sanctioned for violating Rules 3.1, 3.4©, 3.4(e) and 8.4(d)
e. Would it be proper for Martin to have a low‐income employee sit at the counsel table instead of a
company manager to make the company appear nearly broke? Was the incident with Martin‟s secretary
improper? Do any rules apply?
• Model Rule 8.4 ―prejudicial to the administration of justice‖
• Possibly Model Rule 3.4(e):
- ―in trial, allude to any matter that the lawyer does not reasonably believe is relevant or
that will not be supported by admissible evidence…‖
• Possibly Model Rule 3.5(d) engage in conduct intended to disrupt a tribunal.
• Model Rule 3.3(a)(1): lawyer shall not knowingly make a false statement of fact or
law to a tribunal‖
• United States v. Thorean (9th Cir, 1981): switched defendant at counsel table.

33
• Cincinnati Bar Ass’n v. Statzer (Ohio 2003): sanctioned lawyer for intimidating former legal
assistant with false tapes during her deposition.
f. May a lawyer use deception as part of law enforcement or other investigatory functions?
Not usually.
• Model Rule 4.1: no false statements of material fact
• In re: Gatti (Or. 2000): lawyer pretended to be a chiropractor in order to get info about
protocols and guidelines; filed suit against CMR and State Farm.
– Violated equivalent of Model Rules 8.4(c) and 4.1 (received public reprimand)
– Attorney said (govt does this too to ―root out evil‖)
• DC Ethics Opinion (2004): exception for national intelligence agency employees acting in
non‐representational official duties.
g. May a lawyer use deception to help catch a criminal?
• In re Pautler (Colo 2002):
• Prosecutor pretended to be a public defender to get murderer to turn himself in.
– Colo. Supreme Court suspended him for 3 months. (stayed for 12 months of probation).
– No public harm exception to Rule 8.4
a. – Admirable, principled ruling?
h. May a lawyer take advantage of a confidential fax that her adversary inadvertently sent to her? What is
required of attorney who receives them?
Rule 4.4(b)
A lawyer who receives a document relating to the representation of the lawyer‘s client and knows or
reasonably should know that the document was inadvertently sent shall promptly notify the sender.
Comment 2: ―…in order to permit that person to take protective measures.‖
Comment 3: ―…Where a lawyer is not required by applicable law to do so, the decision to voluntarily
return such a document is a matter of professional judgment ordinarily reserved to the lawyer. See Rules
1.2 and 1.4.‖
 ABA Formal Opinion 92‐368 (1992): lawyer should refrain from reading, notify the sending
lawyer and either destroy or return; later withdrawn.
 Whistleblowers:
o – ABA Formal Opinion 94‐382 (1994): refrain from reading, notify, follow instructions,
or wait for conclusive court ruling.
o – Maldonado v. State (DNJ 2004): [discrim case, court disqualified plaintiff‘s attorney
 • Metadata:
 – ABA Formal Opinion 06‐422 (2006): metadata is fair game

7. Client Who Intends to Commit Perjury. Casebook, pp.454-475


a. Rule 3.3
(a) A lawyer shall not knowingly:
(1) make a false statement of fact or law to a tribunal or fail to correct a false statement of
material fact or law previously made to the tribunal by the lawyer;

(3) offer evidence that the lawyer knows to be false. If a lawyer, the lawyer‘s client, or a witness
called by the lawyer, has offered material evidence and the lawyer comes to know of its falsity,
the lawyer shall take reasonable remedial measures, including, if necessary, disclosure to the
tribunal. A lawyer may refuse to offer evidence, other than the testimony of a defendant in a
criminal matter, that the lawyer reasonably believes is false.
Model Rule 3.3, Comment 2:
This Rule sets forth the special duties of lawyers as officers of the court to avoid conduct that

34
undermines the integrity of the adjudicative process. A lawyer acting as an advocate in an
adjudicative proceeding has an obligation to present the client‘s case with persuasive force.
Performance of that duty while maintaining confidences of the client, however, is qualified by
the advocate’s duty of candor to the tribunal. Consequently, although a lawyer in an adversary
proceeding is not required to present an impartial exposition of the law or to vouch for the evidence
submitted in a cause, the lawyer must not allow the tribunal to be misled by false statements of law
or fact or evidence that the lawyer knows to be false..
Comment 5:
Rule ―requires that the lawyer refuse to offer evidence that the lawyer knows to be false, regardless
of the client‘s wishes. This duty is premised on the lawyer‘s obligation as an officer of the court to
prevent the trier of fact from being misled by false evidence...‖
b. Given our norms about loyalty and the constitutional protections of crim. defendants, why isn‟t the
lawyer required to allow the criminal defendant to testify falsely under oath, and then argue to the jury
as if the client‟s perjury is truthful?
• Nix v. Whiteside (1986): No right to commit perjury.
– Seemed to base the constitutional ruling on history of professional ethics standards.
– See Model Rule 3.3, Comment 2
If the lawyer knows the client wants to testify falsely, how should the lawyer proceed?
Model Rule 3.3, Comment 6:
If a lawyer knows that the client intends to testify falsely or wants the lawyer to introduce false evidence,
the lawyer should seek to persuade the client that the evidence should not be offered. If the persuasion is
ineffective and the lawyer continues to represent the client, the lawyer must refuse to offer the false
Evidence. If only a portion of a witness‘s testimony will be false, the lawyer may call the witness to testify
but may not elicit or otherwise permit the witness to present the testimony that the lawyer knows is false.
c. When Smith takes the stand and breaks his promise to be truthful, what should Hawley do? Should Hawley
disclose to the tribunal immediately?
Model Rule 3.3, Comment 10:
―…remonstrate with the client confidentially, advise the client of the lawyer‘s duty of candor to the tribunal
and seek the client‘s cooperation with respect to the withdrawal or correction of the false statements or
evidence. If that fails, the advocate must take further remedial action. If withdrawal from the representation
is not permitted or will not undo the effect of the false evidence, the advocate must make such disclosure to
the tribunal as is reasonably necessary to remedy the situation, even if doing so requires the lawyer to reveal
information that otherwise would be protected by Rule 1.6. It is for the tribunal then to determine what
should be done— making a statement about the matter to the trier of fact, ordering a mistrial or perhaps
nothing.‖
See ABA Formal Opinion 87‐353 (1987):
–Lawyer who knows client has committed perjury must disclose this knowledge to the tribunal if
the lawyer cannot convince the client to rectify it.
Model Rule 3.3, Comment 15:
Normally, a lawyer‘s compliance with the duty of candor imposed by this Rule does not require that the
lawyer withdraw from the representation of a client whose interests will be or have been adversely affected
by the lawyer‘s disclosure.
d. . What if the lawyer discovers that the witness made an immaterial lie under oath but the lawyer has
an argument that the factual misrepresentation is not material?
Model Rule 3.3, (a)(3)
―If a lawyer, the lawyer‘s client, or a witness called by the lawyer, has offered material
evidence and the lawyer comes to know of its falsity, the lawyer shall take reasonable remedial
measures...‖
 Attorney is not allowed to knowingly present any false evidence, but only required to take remedial
measures if the false evidence given was material.
35
– But potentially risky to decide that it is not material.
– Court may disagree about whether something was material
 Is this a good rule? What about the Monroe Freedman argument?
– Trilemma: lawyer must determine all relevant facts; hold confidences; act as officer of the
court, and reveal to court
e. How does an attorney ever really „„know‟‟ that a witness will commit perjury?
Model Rule 1.0(f):
‗‗Knowingly,‘‘ ‗‗known,‘‘ or ‗‗knows‘‘ denotes actual knowledge of the fact in question. A person‘s
knowledge may be inferred from circumstances.
Model Rule 3.3, Comment 8:
―A lawyer‘s reasonable belief that evidence is false does not preclude its presentation to the trier
of fact...although a lawyer should resolve doubts about the veracity of testimony or other
evidence in favor of the client, the lawyer cannot ignore an obvious falsehood.‖
 US v. Midgett (4th Cir., 2003): Not enough that lawyer just doesn‘t believe client.
 ‐Very high standard. Some courts even ―beyond a reasonable doubt‖
f. Client has constitutional right to testify (see Rock v. Arkansas (1987)
See Model Rule 3.3, Comment 9:
Although paragraph (a)(3) only prohibits a lawyer from offering evidence the lawyer knows to be false,
it permits the lawyer to refuse to offer testimony or other proof that the lawyer reasonably believes is
false. Offering such proof may reflect adversely on the lawyer‘s ability to discriminate in the quality of
evidence and thus impair the lawyer‘s effectiveness as an advocate. Because of the special protections
historically provided criminal defendants, however, this Rule does not permit a lawyer to refuse to offer
the testimony of such a client where the lawyer reasonably believes but does not know that the
testimony will be false. Unless the lawyer knows the testimony will be false, the lawyer must honor the
client‘s decision to testify. See also Comment [7].
g. In preparing Smith‟s testimony, if Smith tells Hawley a story that appears untruthful, may Hawley
explain to him the weaknesses that the prosecutor would see in his story?
Thin line re: witness‐coaching; question of fact.
• Resolution Trust Corp. v. Bright (5th Cir., 1993): permissible for lawyers to try to convince witness to
sign an affidavit with favorable version of facts.
– Did the conduct described there seem appropriate? – Where do you draw the line?
a. • Abner v. Elliot (Ohio, 1999): deposition prep crossed the line.
h. Is it permissible to use perjury if the lie comes from government witnesses and its purpose is to trap
criminals? Should it be?
 Not usually.
 • In re Friedman (Ill., 1979) (prosecutor had cop perjure self to collect bribe from defense
attorney)
o – disciplinary action brought against him.
o – compromised the integrity of the courtroom, even if it was for a good cause
 • Operation Greylord? (see Murphy (7th Cir., 1985))
i. Lawyer cannot decide whether a client in a criminal case will take the stand to testify in his own defense
 • Nichols v. Butler (11th Cir., 1992): ineffective assistance of counsel bc lawyer threatened to
withdraw if defendant testified)
 –But, courts may still find ―harmless error‖
j. Is allowing a client who wants to testify falsely to do so in the form of a narrative a good compromise?
• This is the DC Rule and has been allowed in a minority of jurisdictions.
• Pros:
– preserves some of the A‐C relationship,
– allows client to defend self,
– removes lawyer from direct complicity,
36
– signals something is awry
• Cons:
– allows perjury,
a. – can still be pretty prejudicial to defendant.
k. Model Rule 3.3
(a)(3) If a lawyer, the lawyer‘s client, or a witness called by the lawyer, has offered material evidence and
the lawyer comes to know of its falsity, the lawyer shall take reasonable remedial measures, including, if
necessary, disclosure to the tribunal. A lawyer may refuse to offer evidence, other than the testimony of a
defendant in a criminal matter, that the lawyer reasonably believes is false.
(b) A lawyer who represents a client in an adjudicative proceeding and who knows that a person intends to
engage, is engaging or has engaged in criminal or fraudulent conduct related to the proceeding shall take
reasonable remedial measures, including, if necessary, disclosure to the tribunal.
(c) The duties stated in paragraphs (a) and (b) continue to the conclusion of the proceeding, and apply even
if compliance requires disclosure of information otherwise protected by Rule 1.6.

8. The Crusading Prosecutor. Casebook, pp.487-521


a. Publicity
- What limits do the Model Rules place on White‘s use of press conferences and other public
statements to tell the public about his office‘s work?
 Model Rule 3.8
(f) except for statements that are necessary to inform the public of the nature and extent of the
prosecutor‘s action and that serve a legitimate law enforcement purpose, refrain from making
extrajudicial comments that have a substantial likelihood of heightening public condemnation of the
accused and exercise reasonable care to prevent investigators, law enforcement personnel,
employees or other persons assisting or associated with the prosecutor in a criminal case from
making an extrajudicial statement that the prosecutor would be prohibited from making under Rule
3.6 or this Rule.
 Model Rule 3.6(a)
(a) A lawyer who is participating or has participated in the investigation or litigation of a matter shall
not make an extrajudicial statement that the lawyer knows or reasonably should know will be
disseminated by means of public communication and will have a substantial likelihood of materially
prejudicing an adjudicative proceeding in the matter.
NOTE: Some argue that the deck is stacked against defense attorneys
• Most cases involve over‐zealous defense attorneys.
• Shapiro: ―No comment‖ comment is interpreted as a comment and harms defendants.
• In re Morrissey (4th Cir., 1999):
• Few cases with prosecutors being sanctioned for such violations (or for police and other
non‐attorneys‘statements)
• Exceptions:
– Michael Nifong‐ Duke lacrosse case.
– Gansler (Md. 2003) (only issued formal reprimand)
- References to possible future indictments?
• Likely improper: not public record and ―substantial likelihood of materially prejudicing an adjudicative
proceeding in the matter. (but he denies responsibility)
- PP 23, p6 May Gene truthfully respond to the reporter by saying: “the commentator is simply
repeating a charge that one of the suspects has made and the suspect is lying?”
Rule 3.6, Comment 5(1):
Certain factors particularly likely to be prejudicial, including:

37
‗the character, credibility, reputation or criminal record of a party, suspect in a criminal
investigation or witness…‖
Rule 3.6(c):
―fight fire with fire‖ exception possibly applicable, but a stretch:
―lawyer may make a statement that a reasonable lawyer would believe is required to protect a client
from the substantial undue prejudicial effect of recent publicity not initiated by the lawyer or the
lawyer‘s client.‖
NOTE: the fact that Rule 3.6 allows certain types of trial publicity does not preclude other remedies
against lawyers.
- May a prosecutor file a charge that he or she does not believe can be proved beyond a reasonable
doubt? Suppose overcharging would help encourage the defendant to enter a plea to an offense the
defendant clearly committed?
o Under the model rules, yes.
 Prosecutor shall ―refrain from prosecuting a charge that the prosecutor knows is not
supported by probable cause‖ (Model Rule 3.8(a))
BUT ABA Standard Relating to the Prosecution Function 3‐3.9(a): should have
sufficient admissable evidence to support a conviction.
–somewhat higher standard
- Model rules do not require the prosecutor to give exculpatory evidence to the grand jury when
seeking an indictment. There is no constitutional right here.
 Model 3.6(b)
(b) Notwithstanding paragraph (a), a lawyer may state:
(1) the claim, offense or defense involved and, except when prohibited by law, the identity of the
persons involved;
(2) information contained in a public record;
(3) that an investigation of a matter is in progress;
(4) the scheduling or result of any step in litigation;
(5) a request for assistance in obtaining evidence and information necessary thereto;
(6) a warning of danger concerning the behavior of a person involved, when there is reason to
believe that there exists the likelihood of substantial harm to an individual or to the public
interest…
- Should lawyers have more right than non‐lawyers to resist a subpoena to produce records or appear
for questioning?
o Concern over the chilling effect:
- defendants might be less forthright with their counsel if they worried that they would have to
disclose information to a grand jury.
- ―need‖ requirement: makes defense attorneys the last resort.
 ABA Formal Opinion 94‐385: urges resistance to such a subpoena (all ―nonfrivolous claims‖)
 Model Rule 3.8(e):
not subpoena a lawyer in a grand jury or other criminal proceeding to present evidence about a
past or present client unless the prosecutor reasonably believes:
(1) the information sought is not protected from disclosure by any applicable privilege;
(2) the evidence sought is essential to the successful completion of an ongoing
investigation or prosecution; and
(3) no other feasible alternative to obtain the information…
vii. Fee Forfeiture
Cons:
• creates a contingency fee in a criminal case (lawyer only collects if client avoids conviction
on certain charges—can shape attorneys‘ incentives to plea bargain)
• May force litigation against the client (to find ―clean‖ funds)
38
• May need to withdraw because of the unreasonable financial burden
viii. Disclosure
What rules govern the obligations of a prosecutor to provide favorable evidence to the
defense? Should a prosecutor disclose all favorable evidence or just material
exculpatory evidence?
- Brady (and Bagley)– constitutional right to (material) exculpatory evidence relating to
guilt or punishment (―reasonable probability that…the result of the proceeding would have been
different‖)
- ABA Ethics Opinion 09‐454 (Rule 3.8(d) is broader, pertains to all favorable evidence).
RULE 3.8(d):
The prosecutor shall ―make timely disclosure to the defense of all evidence or information known
to the prosecutor that tends to negate the guilt of the accused or mitigates the offense, and, in
connection with sentencing, disclose to the defense and to the tribunal all unprivileged mitigating
information known to the prosecutor, except when the prosecutor is relieved of this
responsibility by a protective order of the tribunal‖
- Consequences can be very serious (disbarment, suspension, dismissed cases).
ix. Cooperation
 Holder Memorandum (privilege waiver, JDAs, and attorney fee advancement).
 Thompson Memorandum (Holder considerations mandatory)
 McNulty Memo (liberalized but broad exceptions remained)
 Stein: due process violation in the KPMG case (govt action).
 Filip Memorandum
- Cooperation now about disclosing all relevant facts, not privilege waiver
- Prosecutors not allowed to request waiver or consider JDAs or fee advancement
x. Should it be improper for a prosecutor to reduce the sanction on a corporate defendant in exchange
for assistance in prosecuting the culpable officers or employees of the corporation?
Pros:
– Easier to identify and punish wrongdoers; bring about justice
– Deterrence
Cons:
– Something unseemly about forcing companies to sell out their employees and officers
– Potential for confused employees to be exploited by their companies and their attorneys.
– Undermines the adversarial system
xi. So far, no court has held life imprisonment to constitute ―substantial bodily harm‖ under Rule
1.6(b)(1).
• Not clear that there would be any duty to the victim not to make her psychiatric studies
public.(PP23, p 16)
 Do you have any duty to the court that tired the case or to the man he believes is wrongly convicted?
o Not under the model rules.
• Rule 3.3: only applies to lawyers involved in the original proceeding and only until the end of the
case.
– BUT doesn‘t prevent disclosure either.
• No obligation to the imprisoned man.
• The ordeals of Alton Logan and others establish this
xii. Story from PP 23. Q from 24.
Does anyone have an obligation to listen to York? Would former defense counsel have such an
obligation, for example? The prosecutor (former or current)?
Model Rules 3.8

39
(g) When a prosecutor knows of new, credible and material evidence creating a reasonable
likelihood that a convicted defendant did not commit an offense of which the defendant was
convicted, the prosecutor shall:
(1) promptly disclose that evidence to an appropriate court or authority, and
(2) if the conviction was obtained in the prosecutor‘s jurisdiction,
(i) promptly disclose that evidence to the defendant unless a court authorizes delay, and
(ii) undertake further investigation, or make reasonable efforts to cause an investigation, to
determine whether the defendant was convicted of an offense that the defendant did not commit.
(h) When a prosecutor knows of clear and convincing evidence establishing that a defendant in
the prosecutor‘s jurisdiction was convicted of an offense that the defendant did not commit, the
prosecutor shall seek to remedy the conviction.
 No formal ethical obligation if the case is over and appeal period finished (Rule 3.3) Rule 1.6 allows
disclosure here (but you may want to go up the ladder first).

9. Marketing Legal Services, Referrals to Specialists. Casebook, pp.522-558


a. What are state‘s interests in regulating attorney communications and advertising?
• Protecting established lawyers; decreasing competition; keeping fees high (cynical view)
• Protecting image of the profession
• Protecting citizens against exploitation and invasions of privacy
• ―stirring up litigation‖ (found to be invalid)
 Supreme Court has played a major role in this area
• Bates v State Bar of AZ (1977)
– Found the state interest unconvincing, esp. the alleged link btw advertising and professionalism
and reputation
– Increases the difficulty of finding ―the lowest‐cost seller of acceptable ability‖
 Advantages established attorneys
- Ruling did not apply to false, deceptive or misleading advertising
• The Supreme Court sets the range of allowed regulation, state bars and courts issue the rules
b. Model Rule 7.1:
―A lawyer shall not make a false or misleading communication about the lawyer or the lawyer‘s
services. A communication is false or misleading if it contains a material misrepresentation of fact or
law, or omits a fact necessary to make the statement considered as a whole not materially
misleading.‖
 Governs all communications about a lawyer‘s services, regardless of medium(see Comment 1)
 Misleading if ―substantial likelihood that it will lead a reasonable person to formulate a
specific conclusion about the lawyer or the lawyer‘s services for which there is no reasonable
factual foundation‖ (Comment 2)
c. Model Rule 7.2:
Allows attorney advertisements through written, recorded or electronic communication.
• Public need for information (especially for unsophisticated, lowerincome)
• Allows basic information (see Comment 2)
Policy (Model Rule 7.2, Comment 3):
―Questions of effectiveness and taste in advertising are matters of speculation and subjective
judgment… Limiting the information that may be advertised… assumes that the bar can accurately
forecast the kind of information that the public would regard as relevant.‖
d. Non‐advertisement communications
– Scholarly articles
– Responding to reporters‘ questions
– Presentations at legal conferences or educational settings
40
– Speeches to public
 Depends on facts. NY Bar found that if primary purpose is to convince audience
to retain the lawyer, then it is considered an advertisement.
e. Nondisclosure of that court fees not included in flat rate
 Is Harrold‘s failure to disclose that clients will have to pay court fees unethical?
Unconstitutional?
- Depends on state rules. States can require him to add a disclaimer re:
- filing fees and other costs (Zauderer, 1985).
 Requiring disclaimers:
 • Depends on the factual context
- – Permissible if to ensure no misrepresentation occurs, but courts sometimes reject states‘
claims about the potential for mispresentation
- – Tillman (11th Cir., 1998): Not permissible for states to require disclaimer warning possible
clients about the consequences of lying
f. Any ethical concerns about his statement that “most of my clients have found that it saves them a
great deal in legal fees…”?
o This is potentially misleading speech that the state may regulate.
Model Rule 7.1, Comment 3:
― …an unsubstantiated comparison of the lawyer‘s services or fees with the services or fees of
other lawyers may be misleading if presented with such specificity as would lead a reasonable
person to conclude that the comparison can be substantiated. The inclusion of an
appropriate disclaimer or qualifying language may preclude a finding that a statement is
likely to create unjustified expectations or otherwise mislead a prospective client
g. What about Harrold‘s writing that some of his former clients agree to act as references?
Model Rule 7.1, Comment 3:
Misleading if ―presented so as to lead a reasonable person to form an unjustified expectation that the
same results could be obtained for other clients in similar matters without reference to the specific
factual and legal circumstances of each client‘s case.‖
Does this cross the line?
 No. Testimonials can be misleading, but the mere fact that they are references does not.
h. Phone Calls to Past and Present Clients
Model Rule 7.3
(a) A lawyer shall not by in‐person, live telephone or real‐time electronic contact solicit professional
employment from a prospective client when a significant motive for the lawyer‘s doing so is the
lawyer‘s pecuniary gain, unless the person contacted:
…(2) has a family, close personal, or prior professional relationship with the lawyer.
- Cannot follow up with plane crash victims families in person
Model Rule 7.3
(b) A lawyer shall not solicit professional employment from a prospective client…if:
(1) the prospective client has made known to the lawyer a desire not to be solicited by the
lawyer; or
(2) the solicitation involves coercion, duress or harassment.
j. Targeted mailing: constitutional issues
• Court treats more like advertisements than solicitation
 Shapero (1988): struck down Kentucky‘s attempt to ban mailings based on specific events.
• less pressure, allows recipient time to reflect
• (some states have disagreed and continued to push the envelope).
– Went For It, Inc. (1995): Florida prevented plaintiffs attorneys (but not defense attorneys) from
targeted direct mail to victims 30 days following an accident or disaster.

41
• Narrower than Shapero, specific population and time window (sparing already traumatized victims
from the pain of seeing lawyer pamphlets.)
k. Policy concerns re: solicitation ban
• ―the lawyer‘s presence and insistence upon being retained immediately.‖ The situation is fraught with
the possibility of undue influence, intimidation, and over‐reaching.
• Prospective clients ―may already feel overwhelmed by the circumstances; ―may find it difficult fully to
evaluate all available alternatives with reasoned judgment and appropriate self‐interest‖ (7.3, Comment
1)
• There are safer alternative means (pre‐recorded, written, etc), which are strongly recommended.
• can be permanently recorded so that they cannot be disputed and may be shared with others.
l. Television Ads
Drive state bars crazy, but serve important purposes
Model Rule 7.2, Comment 3:
―Television is now one of the most powerful media for getting information to the public, particularly
persons of low and moderate income; prohibiting television advertising, therefore, would impede the
flow of information about legal services to many sectors of the public.‖ Similarly, electronic media, such
as the Internet, can be an important source of information about legal services, and lawful
communication by electronic mail is permitted by this Rule.
But see Rule 7.3(a) for the prohibition against the solicitation of a prospective client through a
real‐time electronic exchange that is not initiated by the prospective client.
m. Internet chat‐rooms
Should “real‐time electronic contact” be treated more like in‐person solicitation or letters/advertisements?
• An intermediate medium:
– Real‐time, some pressure to enter agreement,
– but not nearly as coercive (easier for recipient to disengage)

10. Roles and Responsibilities in Law Firms. Casebook, pp.559-574, 594-610


a. Rule 1.1, Comment 2:
A lawyer need not necessarily have special training or prior experience to handle legal problems of a
type with which the lawyer is unfamiliar. A newly admitted lawyer can be as competent as a practitioner
with long experience. Some important legal skills, such as the analysis of precedent, the evaluation of
evidence and legal drafting, are required in all legal problems. Perhaps the most fundamental legal skill
consists of determining what kind of legal problems a situation may involve, a skill that necessarily
transcends any particular specialized knowledge. A lawyer can provide adequate representation in a
wholly novel field through necessary study. Competent representation can also be provided through
the association of a lawyer of established competence in the field in question.
b. Division of Fees PP 25 Can Ramirez refer the case to Castro?
- Yes, if they abide by Model Rule 1.5(e):
 A division of a fee between lawyers who are not in the same firm may be made only if:
(1) the division is in proportion to the services performed by each lawyer or each lawyer
assumes
joint responsibility for the representation;
(2) the client agrees to the arrangement, including the share each lawyer will receive, and the
agreement is confirmed in writing; and
(3) the total fee is reasonable.
NOTE: Technically not allowed to collect ―referral fee‖ but you can ―divide the fee‖
• What are the policy concerns regarding referrals?
– Chasing and selling clients (but so what?)
• What are the benefits?
42
– Clients‘ best interests; safe way to gain experience; possible significant payday for relatively little
work
c. Specialists
Model Rule 7.4
(d) A lawyer shall not state or imply that a lawyer is certified as a specialist in a particular field of law,
unless:
(1) the lawyer has been certified as a specialist by an organization that has been approved
by an appropriate state authority or that has been accredited by the American Bar
Association; and
(2) the name of the certifying organization is clearly identified in the
communication.
Why the concern about lawyers calling themselves specialists?
• Fear that lawyers will play upon the ambiguity of the term to exaggerate their skills and experience.
• States have considerable discretion to set their own standards (but must be even‐handed)
d. Billing for hours never worked
- Unreasonable billing – Rule 1.5(a)
- ―Conduct involving dishonesty, fraud, deceit or misrepresentation‖ Rule 8.4(c)
- Supervisor’s Responsibility
 Model Rule 5.1(b):
 ―A lawyer having direct supervisory authority over another lawyer shall make reasonable efforts
to ensure that the other lawyer conforms to the Rules of Professional Conduct.‖
- Supervisor in example (pp25) was responsible because:
(1) No reasonable efforts taken under 5.1(b).
(2) Also, Model Rule 5.1(c):
A lawyer shall be responsible for another lawyer's violation of the Rules of Professional
Conduct if:
(1) the lawyer orders or, with knowledge of the specific conduct, ratifies the conduct
involved; or
(2) the lawyer is a partner or has comparable managerial authority in the law firm in
which the other lawyer practices, or has direct supervisory authority over the other
lawyer, and knows of the conduct at a time when its consequences can be avoided or
mitigated but fails to take reasonable remedial action.
 Co-managing partner also responsible?
Not quite (no 5.1(c) violation), but still responsible for oversight under Rule 5.1(a).
―A partner in a law firm, and a lawyer who individually or together with other lawyers possesses
comparable managerial authority in a law firm, shall make reasonable efforts to ensure that the
firm has in effect measures giving reasonable assurance that all lawyers in the firm conform to
the Rules of Professional Conduct.‖
Model Rule 5.1, Comment 2
―[Rule] requires lawyers with managerial authority…to make reasonable efforts to establish
internal policies and procedures designed to provide reasonable assurance... Such policies and
procedures include those designed to detect and resolve conflicts of interest, identify dates by
which actions must be taken in pending matters, account for client funds and property and
ensure that inexperienced lawyers are properly supervised.‖
Depends on the organizational context (see Comment 3)
e. Responsibility if instructed by superiors
- Would Able‟s liability be different if he had been instructed to overbill by his superiors?
Model Rule 5.2, Comment 1:
―a lawyer is not relieved of responsibility for a violation by the fact that the lawyer acted at the
direction of a supervisor‖
43
BUT
Model Rule 5.2(b):
―A subordinate lawyer does not violate the Rules of Professional Conduct if that lawyer acts in
accordance with a supervisory lawyer‘s reasonable resolution of an arguable question of professional
duty.‖
Nothing arguable about attorney‘s duty not to bill fraudulently.
f. If a lawyer refuses a direction to do something illegal, does that lawyer have any protection against
being fired for it?
• General rule: clients can fire lawyers for any reason they want.
– Question is whether they can recover damages
– Early cases (Illinois): lawyers cannot recover damages
– Other courts have mostly refused to follow (incentives)
• What about for law firm associates fired by partners?
– Cases mixed: Wider and Lichtman v. Jacobson and Bohatch

11. Unauthorized Practice, MJD practices, and Ancillary Legal Services. Casebook, pp.630-
654
a. ―Voluntary‖ pro‐bono work
Model Rule 6.1
Every lawyer has a professional responsibility to provide legal services to those unable to pay. A lawyer
should aspire to render at least (50) hours of pro bono publico legal services per year.‖
Substantial majority of pro bono hours should be devoted to:
(1) persons of limited means or
(2) charitable, religious, civic, community, governmental and educational organizations in matters that
are designed primarily to address the needs of persons of limited means; and
- Also extends to parties ―seeking to secure or protect civil rights, civil liberties or public rights‖ and
includes
 ―activities for improving the law‖ (b(3))
Why should lawyers have pro bono responsibilities when other professionals do not?
o • They are officers of the court
o • It is necessary for adversarial system to work
o • It can inform the public, increase public faith in the profession
o • Moral obligation?
- Although some attorneys have personal incentives, this is essentially a tax
b. Appointment by tribunal
Model Rule 6.2
A lawyer shall not seek to avoid appointment by a tribunal to represent a person except for good cause,
such as:
(a) representing the client is likely to result in violation of the Rules of Professional Conduct or other
law;
(b) representing the client is likely to result in an unreasonable financial burden on the lawyer; or
(c) the client or the cause is so repugnant to the lawyer as to be likely to impair the client‐lawyer
relationship or the lawyer‘s ability to represent the client.
What is the basis of the should/shall distinction between Model Rules 6.1 and 6.2? Do cases holding that
defendants have a constitutional right to counsel necessarily imply that attorneys must bear the
uncompensated burden of such representation? Is this distinction principled?
Tradition and the aura of the constitutional rights, perhaps.
c. Limitations on Court Appointments
i. Important source of ethical issues.

44
ii. Courts not allowed to impose undue burdens or unreasonable limits on counsel:
 Olive v. Maas (Fla.2002): Cap on post‐conviction relief in capital cases unconstitutional;
interferes with right to counsel.
 Ferri v. Ackerman (1979) (indigent federal criminal defendant may file malpractice suit against
appointed counsel).
 Zarabia v. Bradshaw, 912 P.2d 5 (Ariz.1996): system for appointing counsel from a pool that
included lawyers with no experience in either criminal law or trial work unconstitutional.
iii. Should counsel be appointed in civil suits? What are the pros and cons?
- Mallard (1989): limits on court authority to appoint counsel in many civil cases (can only
―request‖).
Why not develop a more robust publicly‐funded system, like with medicine/healthcare?
 • Legal Services Corporation (LSC), federal agency
 • Govt. money can come with limitations
d. The practice of law
- ―A lawyer shall not practice law in a jurisdiction in violation of the regulation of the legal profession
in that jurisdiction, or assist another in doing so.‖ (Model Rule 5.5(a))
 But what is the practice of law?
- The definition very vague. ABA Task Force considered listing specific tasks but ltimately
left it to the individual states.
- Definition is ―established by law and varies from one jurisdiction to another‖ (Model
Rule 5.5, comment 2).
- Model Code EC 3‐5: services that requires attorneys‘ ―professional judgment,‖ applying
the law to the specific legal problem of a client.
e. Unlicensed Practice of Law
- Why such strict regulations of the unlicensed practice of law?
 Endangers clients, threatens the adversarial system, makes it harder for courts to regulate the
profession, takes $$ from attorneys
 Then why do we let people act pro se, representing themselves in court or in negotiations, if
unlicensed practice is so terrible?
 It is a narrow exception that ultimately undermines the rule.
 Books are usually not unlicensed self-practice of law
f. Filling in the Blanks
- Generally does not constitute the unlicensed practice of law
• As a general matter, entering factual information into forms is not legal work, per se.
• Some caselaw to the contrary (see Glasgow (Tenn.App.1999) (helping clients fill in blanks on
divorce and real estate forms was UPL)) but courts/bar assocations seem to be moving away from
this.
• Condra (Ind.2007)(no legal advice was involved; filling in the blanks was well within the skill set
of the nonlawyers)
• Kentucky state bar issued advisory opinion arguing that performance of a real estate closing was
unauthorized practice of law; Kentucky court rejected this.
g. Computer-assisted drafting programs
- Courts have often treated this as practice of law.
- • Parsons Technology (N.D.Tex.1999): overturned – Quicken Family Lawyer program‐‐UPL
- • Kistler (9th Cir.2007): program took client facts, selected and completed bankruptcy forms—UPL
- • Some public backlash
 – TX decision overturned; clearly a demand for these products
h. Representing outside of your licensed jurisdiction
- Model Rule 5.5c:
- A lawyer…may provide legal services on a temporary basis in this jurisdiction that:
45
- (1) are undertaken in association with a lawyer who is admitted to practice…;
- (2) are in or reasonably related to a pending or potential proceeding before a tribunal …,
if…authorized by law or order to appear in such proceeding or reasonably expects to be so
authorized;
- (3) …if the services arise out of or are reasonably related to the lawyer‘s practice in a jurisdiction in
which the lawyer is admitted to practice and are not services for which the forum requires pro hac
vice admission; or
- (4) are not within paragraphs (c)(2) or (c)(3) and arise out of or are reasonably related to the lawyer‘s
practice in a jurisdiction in which the lawyer is admitted to practice.
- If expecting to appear before a tribunal, pro hac vice usually required, under 5.5c(2)
- This is potentially very expansive, very subjective (see 5.5c(4)).
- Federal law preempts state rules in some instances: tax preparation, FDIC savings & loan legal
documents; patent filing and research; pension planning
 – See Model Rule 5.5(d)(2) (―services that the lawyer is authorized by federal or other law
to provide in this jurisdiction‖).
i. Partnerships with Non-Attorneys
- Model Rules very concerned about attorneys working with non‐attorneys.
- Model Rule 5.4
- (b) A lawyer shall not form a partnership with a nonlawyer if any of the activities of the partnership
consist of the practice of law.
- (c) A lawyer shall not permit a person who recommends, employs, or pays the lawyer to render legal
services for another to direct or regulate the lawyer‘s professional judgment in rendering such legal
services.
- Why the concern? Is it justifiable?
- Concern is about lawyers‘ independent judgment being controlled and compromised (see Comment
2).
- Also, concerns about disciplinary authority
j. Ban on corporate practice of law
- Model Rule 5.4(d)
- A lawyer shall not practice with or in the form of a professional corporation or association
authorized to practice law for a profit, if:
- (1) a nonlawyer owns any interest therein, except that a fiduciary representative of the estate of a
lawyer may hold the stock or interest of the lawyer for a reasonable time during administration;
- (2) a nonlawyer is a corporate director or officer thereof or occupies the position of similar
responsibility…; or
- (3) a nonlawyer has the right to direct or control the professional judgment of a lawyer.
- Should courts be concerned if Dowd, a licensed lawyer, does general estate planning work for the
customers of an insurance company?
- Debatable in our case. If Dowd is ―practicing with the corporation,‖ he is in trouble. But he could
possibly argue that he just helps the company when it needs his legal advice. Plausible?
k. Full Time Representatives for Company Policyholders
- What about insurance companies who hire lawyers as full‐time employees to represent the
company‟s policyholders in actions brought by third parties? Is that simply a more efficient way to
- pay the lawyers?
 If lawyer is employee of the insurance company and is in that role providing legal services to
third party, seems like a violation. Many states allow this though.
l. One-Stop Shops
- How do these principles apply to the practice of accounting firms hiring lawyers full‐time to give
corporate and transactional advice to the firms‟ business clients? What should be the content of an
ethics rule dealing with „„one‐stop business services‟‟ by accounting firms?
46
- • The ABA Commission on Multidisciplinary Practice (MDP) issued favorable report in 1999 and
proposed a new rule to regulate such firms.
- • ABA rejected, but directed the ABA Ethics Committee to investigate‗‗sideby‐side‘‘ relationships
between law firms and other service providers.
- The biggest sticking point involves conflict of interest rules. This would eliminate all of the major
accounting firms.
m. Law Firm Based?
- Could Dowd eliminate all problems with his multidisciplinary organization if he simply ran it out of
- his law firm?
- Quite possibly, if he abides by the restrictions of Rule 5.4.
- Law firms can hire accountants, insurance specialists, lobbyists as employees, just not as equal
partners.

47

Anda mungkin juga menyukai